Sunteți pe pagina 1din 69

JURISDICTION CASES:

City of Lapu Lapu vs. PEZA


G.R. no. 184203
Facts:
1. In 1995, the PEZA was created by virtue of Republic Act No. 7916 or “the
Special Economic Zone Act of 1995” to operate, administer, manage, and
develop economic zones in the country. The PEZA was granted the power to
register, regulate, and supervise the enterprises located in the economic
zones. By virtue of the law, the export processing zone in Mariveles, Bataan
became the Bataan Economic Zone and the Mactan Export Processing
Zone the Mactan Economic Zone.
2. The City contends that due to the enactment of the LGC, specifically
withdrawing all tax exemptions and with the PEZA law of 1995 which did
not have any provisions on tax exemptions, it maintains that PEZA is liable
for real property tax.
Issue: Whether or not PEZA should be exempted from real property taxation.
Held: Yes. Under Section 234(a) of the Local Government Code, real properties
owned by the Republic of the Philippines are exempt from real property
taxes. Properties owned by the state are either property of public dominion or
patrimonial property as per Art. 420.
Citing Manila International Airport Authority: Properties of public dominion,
being for public use, are not subject to levy, encumbrance or disposition through
public or private sale. Any encumbrance, levy on execution or auction sale of any
property of public dominion is void for being contrary to public policy. Essential
public services will stop if properties of public dominion are subject to
encumbrances, foreclosures and auction sale.
In this case, the properties sought to be taxed are located in publicly owned
economic zones. These economic zones are property of public dominion – sites
which were reserved by President Marcos under Proclamation No. 1811, Series of
1979 (Mactan).
Petition Denied.

MANUEL LUIS C. GONZALES v. GJH LAND, GR No. 202664, 2015-11-20


Facts:
(respondents) before the RTC of
Muntinlupa City seeking to enjoin the sale of S.J. Land, Inc.'s shares... which they
purportedly bought from S.J. Global, Inc.
petitioners alleged that the subscriptions for the said shares were already paid by
them in full in the books of S.J.
Land, Inc.,... but were nonetheless offered for sale on July 29, 2011 to the
corporation's stockholders
Branch 276, which is not a Special Commercial Court.
issued a temporary restraining order... and later... granted the application for a writ
of preliminary injunction.
respondents filed a motion to dismiss... on the ground of lack of jurisdiction over
the subject matter, pointing out that the case involves an intra-corporate dispute
and... should, thus, be heard by the designated Special Commercial Court of
Muntinlupa City.
April 17, 2012, Branch 276 granted the motion to dismiss filed by respondents. It
found that the case involves an intra-corporate dispute
It pointed out that the RTC of Muntinlupa City, Branch 256 (Branch 256) was
specifically designated by the Court as the Special Commercial Court, hence,
Branch 276 had no jurisdiction over the case and cannot lawfully exercise
jurisdiction on the matter,... including the issuance of a Writ of Preliminary
Injunction... petitioners filed a motion for reconsideration,... Branch 276 denied
the motion for reconsideration, holding that it has no authority or power to order
the transfer of the case to the proper Special Commercial Court
Issues:
The essential issue for the Court's resolution is whether or not Branch 276 of the
RTC of Muntinlupa City erred in dismissing the case for lack of jurisdiction over
the subject matter.
Ruling:
WHEREFORE, the petition is GRANTED. The Orders dated April 17, 2012 and
July 9, 2012 of the Regional Trial Court (RTC) of Muntinlupa City, Branch 276 in
Civil Case No. 11-077 are hereby REVERSED and SET ASIDE. Civil Case No.
11-077 is REFERRED... to the Executive Judge of the RTC of Muntinlupa City
for re-docketing as a commercial case. Thereafter, the Executive Judge shall
ASSIGN said case to Branch 256, the sole designated Special Commercial Court
in the RTC of Muntinlupa City, which is ORDERED to resolve... the case with
reasonable dispatch. In this regard, the Clerk of Court of said RTC shall
DETERMINE the appropriate amount of docket fees and, in so doing, ORDER the
payment of any difference or, on the other hand, refund any excess.

CRESPO VS MOGUL CASE DIGEST


FACTS:

Petitioner Mario Crespo was accused for Estafa in the Circuit Criminal Court of
Lucena City. When the case was set for arraignment, the accused filed a motion
for defer arraignment on the ground that there was a pending petition for review
filed with the Secretary of Justice. However, Justice Mogul denied the motion, but
the arraignment was deferred in a much later date to afford time for the petitioner
to elevate the mater to the appellate court.
The accused filed a petition for certiorari and prohibition with prayer for a
preliminary writ of injunction to the CA. The CA ordered the trial court to refrain
from proceeding with the arraignment until further orders of the Court.
Undersecretary of Justice, Hon. Catalino Macaraig Jr., resolved the petition for
review reversed the resolution of the office of the Provincial Fiscal and directed
the Fiscal to move for immediate dismissal of the information filed against the
accused. Judge Mogul denied the motion for dismissal of the case ad set the
arraignment. The accused then filed a petition for Certiorari, prohibition and
mandamus with petition for the issuance of preliminary writ of prohibition and/or
temporary restraining order in the CA. The CA dismissed the order and lifted the
restraining order.
Issue: Whether the trial court may refuse to grant a motion to dismiss filed by the
Fiscal under orders fro, the Secretary of Justice and insists on arraignment and trial
on the merits.

HELD:
It is a cardinal principle that all criminal actions either commenced by complaint
or by information shall be prosecuted under the direction and control of the fiscal.
17 The institution of a criminal action depends upon the sound discretion of the
fiscal. The reason for placing the criminal prosecution under the direction and
control of the fiscal is to prevent malicious or unfounded prosecution by private
persons. 19 It cannot be controlled by the complainant.

However, the action of the fiscal or prosecutor is not without any limitation or
control. The same is subject to the approval of the provincial or city fiscal or the
chief state prosecutor as the case maybe and it maybe elevated for review to the
Secretary of Justice who has the power to affirm, modify or reverse the action or
opinion of the fiscal. Consequently the Secretary of Justice may direct that a
motion to dismiss the case be filed in Court or otherwise, that an information be
filed in Court.
The filing of a complaint or information in Court initiates a criminal action. The
Court thereby acquires jurisdiction over the case, which is the authority to hear
and determine the case. The preliminary investigation conducted by the fiscal for
the purpose of determining whether a prima facie case exists warranting the
prosecution of the accused is terminated upon the filing of the information in the
proper court.
LAND BANK OF THE GR No. 180384
PHILIPPINES v. VILLEGAS
March 26, 2010 Abad, J.
TOPIC IN SYLLABUS: Jurisdiction
SUMMARY: LBP filed cases for determination of just
compensation against Villegas and heirs of Noel before
RTC Br. 32 of Dumaguete City, the designated SAC of
Negros Oriental. The subject lands were located in
Guihulngan City and Bayawan City, which are outside
the regular territorial jurisdiction of RTC Br. 32. As
such, RTC, Br. 32 dismissed both cases. LBP’s MRs
were also dismissed, prompting it to file petitions for
certiorari before the SC. SC ruled that RTC Br. 32 has
jurisdiction over all cases of determination of just
compensation within the province of Negros Oriental,
regardless of WoN the properties are outside its regular
jurisdiction.
DOCTRINE: By “special” jurisdiction, SACs exercise
power in addition to or over and above the ordinary
jurisdiction of the RTC, such as taking cognizance of
suits involving agricultural lands located outside their
regular territorial jurisdiction, so long as they are within
the province where they sit as SACs.

FACTS:
Land Bank (LBP) filed cases for determination of just compensation against
Corazon Villegas, and heirs of Catalino Noel and Procula Sy before the RTC of
Dumaguete City Br.32, sitting as a Special Agrarian Court (SAC) for the province
of Negros Oriental. Villegas’ property was in Guihulngan City, while the heirs’
land was in Bayawan City, both in Negros Oriental, but which were outside the
regular territorial jurisdiction of RTC Br. 32 of Dumaguete City.

RTC Br. 32 dismissed both cases for lack of jurisdiction, ruling that, although it
was designated as the SAC for Negros Oriental, such designation did not expand
its territorial jurisdiction to hear agrarian cases, which are within the territorial
jurisdiction of Guihulngan City and Bayawan City.
LBP moved for reconsideration of the 2 cases but Br. 32 dismissed the motions.
LBP directly filed petitions for certiorari before the SC, raising purely a question
of law.

RESPONDENT’S ARGUMENT(S): RTC, Br. 32 based its order on Deputy


Court Administrator Zenaida Elepaño’s opinion that single sala courts have
jurisdiction over agrarian cases involving lands located within its territorial
jurisdiction. Furthermore, the RTC’s designation as a SAC did not expand its
territorial jurisdiction. The jurisdiction of such SAC over agrarian cases is co-
extensive with its territorial jurisdiction. While it has been designated as SAC for
the Province of Negros Oriental, its jurisdiction as an RTC did not cover the whole
province. As for the heirs of Noel, LBP had already paid them for their land.

ISSUE(S): WoN an RTC, acting as Special Agrarian Court, has jurisdiction


over just compensation cases involving agricultural lands located outside its
regular jurisdiction but within the province where it is designated as an
agrarian court under the Comprehensive Agrarian Reform Law of 1998 —Y

HELD:
Jurisdiction is the court’s authority to hear and determine a case. Such jurisdiction
over the nature and subject matter of an action is conferred by law.

In the case at bar, Secs. 56 and 57 of the CARL of 1998 provide that a branch of
an RTC designated as SAC for a province has original and exclusive jurisdiction
over all petitions for the determination of just compensation and prosecution of all
criminal offenses under CARL in that province (Republic v. CA).

By “special” jurisdiction, SACs exercise power in addition to or over and above


the ordinary jurisdiction of the RTC, such as taking cognizance of suits involving
agricultural lands located outside their regular territorial jurisdiction, so long as
they are within the province where they sit as SACs.

CARL requires the designation of the SC before an RTC branch can function as a
SAC. Br. 64 of Guihulngan City and Br. 63 of Bayawan City have not been
designated as SAC by the SC. Consequently, they cannot hear just compensation
just because the lands subject of such cases happen to be within their territorial
jurisdiction. Since RTC Br. 32 of Dumaguete City is the designated SAC for
Negros Oriental, it has jurisdiction over all cases of determination of just
compensation involving agricultural lands within that province, regardless of WoN
those properties are outside its regular territorial jurisdiction.

SC GRANTS the petitions and SETS ASIDE the orders of RTC Br. 32 of
Dumaguete City. The Court DIRECTS said RTC to immediately hear and decide
the 2 cases unless a compromise agreement has been approved.

NOTE: The scra states that it’s a Special Civil Action for certiorari (R65), but
it was mentioned in the body that the petition for certiorari was directly filed
with the SC since it raised only questions of law (R45).

TIJAM vs. SIBONGHANOY Question on CFI's Jurisdiction) 23 SCRA 29


TIJAM vs. SIBONGHANOY (23 SCRA 29)
(Question on Court of First Instance's* Jurisdiction)
Digested Case

FACTS:
Tijam filed for recovery of P1,908 + legal interest from Sibongahanoy. Defendants
filed a counter bond with Manila Surety and Fidelity Co (Surety). Judgement was
in favour of the plaintiffs, a writ of execution was issued against the defendant.
Defendants moved for writ of execution against surety which was granted. Surety
moved to quash the writ but was denied, appealed to CA without raising the issue
on lack of jurisdiction.

CA affirmed the appealed decision. Surety then filed Motion to Dismiss on the
ground of lack of jurisdiction against CFI Cebu in view of the effectivity of
Judiciary Act of 1948 a month before the filing of the petition for recovery. Act
placed original exclusive jurisdiction of inferior courts all civil actions for
demands not exceeding 2,000 exclusive of interest. CA set aside its earlier
decision and referred the case to SC since it has exclusive jurisdiction over "all
cases in which the jurisdiction of any inferior court is in issue.

ISSUE:
WON Surety bond is estopped from questioning the jurisdiction of the CFI Cebu
for the first time upon appeal.

HELD:
YES, SC believes that that the Surety is now barred by laches from invoking this
plea after almost fifteen years before the Surety filed its motion to dismiss raising
the question of lack of jurisdiction for the first time - A party may be estopped or
barred from raising a question in different ways and for different reasons. Thus we
speak of estoppel in pais, or estoppel by deed or by record, and of estoppel by
laches. Laches, in a general sense is failure or neglect, for an unreasonable and
unexplained length of time, to do that which, by exercising due diligence, could or
should have been done earlier - Furthermore, it has also been held that after
voluntarily submitting a cause and encountering an adverse decision on the merits,
it is too late for the loser to question the jurisdiction or power of the court -
"undesirable practice" of a party submitting his case for decision and then
accepting the judgment, only if favorable, and attacking it for lack of jurisdiction,
when adverse.

: Other merits on the appeal : The surety insists that the lower court should have
granted its motion to quash the writ of execution because the same was issued
without the summary hearing - Summary hearing is "not intended to be carried on
in the formal manner in which ordinary actions are prosecuted" (83 C.J.S. 792). It
is, rather, a procedure by which a question is resolved "with dispatch, with the
least possible delay, and in preference to ordinary legal and regular judicial
proceedings" (Ibid, p. 790). What is essential is that "the defendant is notified or
summoned to appear and is given an opportunity to hear what is urged upon him,
and to interpose a defense, after which follows an adjudication of the rights of the
parties - In the case at bar, the surety had been notified of the plaintiffs' motion for
execution and of the date when the same would be submitted for consideration. In
fact, the surety's counsel was present in court when the motion was called, and it
was upon his request that the court a quo gave him a period of four days within
which to file an answer. Yet he allowed that period to lapse without filing an
answer or objection. The surety cannot now, therefore, complain that it was
deprived of its day in court.

The orders appealed from are affirmed.

CaseDig: Vda de Herrera vs. Bernardo


G.R. No. 170251, 01 June 2011
Posted by Rio Vie C. Dumalag on 25 July 2018

FACTS:

Emelita Bernardo, representing the heirs of Crisanto Bernardo, filed a complaint


against Alfredo Herrera for unlawful claim, interference, disturbance, harassment
and trespassing before the Commission of the Settlement of Land Problem
(referred hereafter as COSLAP) over 7,993-square meter portion of land. The land
was claimed by the respondents to be owned by their predecessor Crisanto
Bernardo and was acquired later by Crisanto S. Bernardo and covered by Tax
Declaration CD-006-0828 under their name.

Celia Vda de Herrera alleged that the 700-square meter portion of the disputed
land was brought by Diosdado Herrera, father of her (late) husband Alfredo, from
a Domingo Villaran. Alfredo inherited the property upon his father's death.

COSLAP ruled in its 6 December 1999 decision in favor of the Bernardos. Alfredo
filed a motion of reconsideration about the said decision and to reopen the
proceedings, but COSLAP denied his motion in its 21 August 2002 and 6
December 1999 orders. Alfredo's surviving spouse Celia, filed a petition for
certiorari with the CA. However, the CA's 12th division affirmed COSLAP's
decision as stated in its 28 April 2005 decision.

The CA ruled that COSLAP has exclusive jurisdiction over the land dispute, and
even if assumingly, COSLAP does not have jurisdiction over the said case, Celia
is estopped to question COSLAP's jurisdiction on the grounds that first, her
husband failed to raise the issue of jurisdiction before that body and second, he
actively participated in the proceedings. Celia filed her motion of reconsideration
but the CA denied that through its 17 October 2005 resolution.

ISSUE:

Whether or not COSLAP has jurisdiction over the ownership case of the land
disputed by Herreras and Bernardos.

HELD:

NO, as their dispute does not fall under situation mentioned in Sec. 3 of E.O. 561.
Administrative agencies, like the COSLAP, are tribunals of limited jurisdiction
that can only wield powers which are specifically granted to it by its enabling
statute. Under Section 3 of E.O. No. 561, the COSLAP has two options in acting
on a land dispute or problem lodged before it, to wit: (a) refer the matter to the
agency having appropriate jurisdiction for settlement/resolution; or (b) assume
jurisdiction if the matter is one of those enumerated in paragraph 2 (a) to (e) of the
law, if such case is critical and explosive in nature, taking into account the large
number of parties involved, the presence or emergence of social unrest, or other
similar critical situations requiring immediate action. In resolving whether to
assume jurisdiction over a case or to refer the same to the particular agency
concerned, the COSLAP has to consider the nature or classification of the land
involved, the parties to the case, the nature of the questions raised, and the need
for immediate and urgent action thereon to prevent injuries to persons and damage
or destruction to property. The law does not vest jurisdiction on the COSLAP over
any land dispute or problem.

Since the COSLAP has no jurisdiction over the action, all the proceedings therein,
including the decision rendered, are null and void. A judgment issued by a quasi-
judicial body without jurisdiction is void. It cannot be the source of any right or
create any obligation. All acts performed pursuant to it and all claims emanating
from it have no legal effect. Having no legal effect, the situation is the same as it
would be as if there was no judgment at all. It leaves the parties in the position
they were before the proceedings.

Respondents' allegation that petitioner is estopped from questioning the


jurisdiction of the COSLAP by reason of laches does not hold water. Petitioner is
not estopped from raising the jurisdictional issue, because it may be raised at any
stage of the proceedings, even on appeal, and is not lost by waiver or by estoppel.
The fact that a person attempts to invoke unauthorized jurisdiction of a court does
not estop him from thereafter challenging its jurisdiction over the subject matter,
since such jurisdiction must arise by law and not by mere consent of the parties.
Office of the Ombudsman v. Rodriguez | ema
July 23, 2010
OFFICE OF THE OMBUDSMAN, petitioner, vs.
ROLSON RODRIGUEZ, respondent.
CARPIO, J.:

SUMMARY: Two complaints for abuse of authority, dishonesty,


misconduct, and neglect were filed against P/B Rodriguez, one in the
Sangguniang Bayan (SB) of Binalbagan, Negros Occidental, and another
in the Ombudsman. The complaint before the Ombudsman was filed on
Aug. 26, 2003; complaint in the SB was filed Sept. 1. SB served notice on
Rodriguez on Sept. 8, while Ombudsman did so on Sept. 10. After several
motions filed, [allegation of forum shopping, motion to dismiss filed by
Rodriguez] the complainants eventually withdrew the SB complaint so they
could focus on the complaint with the Ombudsman. In the Ombudsman
proceeding, Rodriguez filed MTD, claiming that the SB still had jurisdiction
because he never received a decision or resolution dismissing that
complaint. Complainants argued that the case was dismissed after the
Vice-Mayor granted their motion to withdraw. Rodriguez replied that the
dismissal was invalid because only the vice-mayor signed it. Later,
Ombudsman rendered a decision dismissing Rodriguez from his position,
disqualifying him from public office, and forfeiting his benefits and CSC
eligibility. On appeal, CA reversed, holding that SB still had jurisdiction
because it was the first to serve notice on Rodriguez. On appeal by the
Ombudsman, SC reversed CA and affirmed the Ombudsman decision,
ruling that the Ombudsman had concurrent jurisdiction with the SB under
RA 8749 and the LGC, because Rodriguez is a punong barangay (SG 14).
Contrary to CA conclusion and Rodriguez’ contention that jurisdiction is
acquired by service of summons [thus SB acquired jurisdiction first], in
cases of 2 agencies exercising concurrent jurisdiction, the body in w/c the
complaint is filed first, and which opts to take cognizance of the case,
acquires jurisdiction to the exclusion of other tribunals exercising
concurrent jurisdiction, and the jurisdiction continues until the case is
terminated. CAB: Complaint with the Ombudsman was filed first, so when
it took of cognizance of [opted to assume jurisdiction over] the case,
jurisdiction had already vested, to the exclusion of the SB. Ombudsman
decision was thus rendered with jurisdiction and should be upheld.
DOCTRINE: Under LGC 60, the sangguniang bayan has no power to
remove an elective barangay official. Apart from the Ombudsman, only a
proper court may do so. Unlike the sangguniang bayan, the powers of the
Ombudsman are not merely recommendatory. The Ombudsman is clothed
with authority to directly remove an erring public official other than
members of Congress and the Judiciary who may be removed only by
impeachment. Ombudsman has concurrent jurisdiction with the
sangguniang bayan over administrative cases against elective barangay
officials with salary grade below 27, e.g., punong barangay. The rule
against forum shopping applies only to judicial cases or proceedings, not
to administrative cases.

NATURE: Petition for review under ROC 45. Administrative complaint filed
with the Ombudsman.

FACTS
 Aug. 26, 2003 – Ombudsman for Visayas (OMB) received a
complaint against P/B Rolson RODRIGUEZ of Sto. Rosario,
Binalbagan, Negros Occidental. The complaint alleged abuse of
authority, dishonesty, oppression, misconduct in office, and neglect of
duty.
 Sep. 1, 2003 – Sangguniang Bayan (SB) of Binalbagan, through
Vice-Mayor Jose G. YULO, received a similar complaint against
Rodriguez. [no statement as to who the complainants were]
 Sep. 8, 2003 – Yulo issued a notice ordering Rodriguez to file an
answer within 15 days from receipt of such notice.
 Sep, 10, 2003 – OMB required Rodriguez to file his answer.
 Sep. 23, 2003 – Rodriguez moved to dismiss the complaint in the SB
for being baseless in fact and in law. He also argued that the
complainants violated the rule against forum shopping.
 Oct. 24, 2003 – Rodriguez moved to dismiss the OMB complaint on
the grounds of litis pendentia and forum shopping, arguing that the
SB had acquired jurisdiction on Sep. 8.
 Complainants, through counsel, moved to withdraw the SB complaint
to prioritize the OMB complaint.
o Rodriguez insisted that the SB complaint be dismissed on the
ground of forum shopping
o Complainants admitted to forum shopping (LOL) and claimed
that they were not assisted by counsel when they filed the
complaint.
 Nov. 4, 2003 – Yulo dismissed the SB complaint.
 Jan. 29, 2004 – OMB ordered both parties to file position papers.
Rodriguez filed MR, citing pendency of his MTD.
o MR denied. MTD is a prohibited pleading under AO 17, Rule III,
Sec.5(g). OMB: Magfile ka ng position paper.
o Rodriguez: Sige fa-file ako. SB still had jurisdiction over his
person because he has not received any resolution or decision
indicating the dismissal of the SB case.
o Complainants: There was no more complaint in the SB because
Vice-Mayor Yulo granted their motion to withdraw.
o Rodriguez’ rejoinder: Dismissal not valid because only the Vice-
Mayor signed it.
 Sep. 21, 2004 – OMBUDSMAN DECISION
o Found Rodriguez GUILTY of dishonesty and oppression
o Rodriguez dismissed from service, w/ forfeiture of all benefits
and civil service eligibilites, and disqualification from public
office.
o Rodriguez filed MR.
 Jan 12, 2005 – OMB directed the Mayor of Binalbagan to dismiss
Rodriguez.
 Rodriguez filed a petition for review with the CA.
 May 8, 2006 – CA DECISION
o OMB decision set aside for lack of jurisdiction
o Directed SB to continue hearing the case, as it acquired primary
jurisdiction over Rodriguez, to the exclusion of the OMB.
o BASIS: RoC 46, Sec. 4. SB was the first to serve notice on
Rodriguez.
o OMB filed the present petition
 OMB: Jurisdiction over the person is acquired once a body vested
with jurisdiction takes cognizance of the complaint. OMB was first to
take cognizance of the complaint because the SB complaint was filed
later. Summons or notices do not vest jurisdiction over the person in
an administrative case. Consistent with the rule on concurrent
jurisdiction, OMB exercise of jurisdiction must be to the exclusion of
the SB.
 Rodriguez: When a competent body has acquired jurisdiction over a
complaint and over the person of the respondent, other bodies are
excluded from exercising jurisdiction over the same complaint. LGC
IRR, Art. 124 provides that an elective official may be removed by the
proper court or by disciplining authority whichever acquires
jurisdiction first to the exclusion of the other. SB acquired jurisdiction
first. Jurisdiction in administrative cases is acquired by service of
summons or other compulsory processes. Complainants committed
forum shopping when they filed two identical complaints in two
disciplining authorities exercising concurrent jurisdiction.

ISSUES (HELD)
1) W/N the complainants violated the rule against forum shopping when
they filed in the Ombudsman and the sangguniang bayan identical
complaints against Rodriguez (NO)
2) Who acquired jurisdiction first - the sangguniang bayan or the
Ombudsman? (Ombudsman)

RATIO
1) FORUM SHOPPING NOT APPLICABLE IN ADMINISTRATIVE CASES
“The facts in this case are analogous to those in Laxina, Sr. v.
Ombudsman, which likewise involved identical administrative complaints
filed in both the Ombudsman and the sangguniang panlungsod against a
punong barangay for grave misconduct. The Court held therein that the
rule against forum shopping applied only to judicial cases or
proceedings, not to administrative cases. Thus, even if complainants
filed in the Ombudsman and the sangguniang bayan identical complaints
against [Rodriguez], they did not violate the rule against forum shopping
because their complaint was in the nature of an administrative case.”

2) OMBUDSMAN HAS CONCURRENT JURISDICTION WITH SB BUT


COMPLAINT WAS FIRST FILED WITH OMBUDSMAN
 Constitution, Art. XI, Sec. 13(1): The Ombudsman shall have the
following powers, functions, and duties: (1) Investigate on its own,
or on complaint by any person, any act or omission of any public
official, employee, office, or agency, when such act or omission
appears to be illegal, unjust, improper, or inefficient.
 Ombudsman Act, Sec. 15: The Ombudsman shall have the following
powers, functions, and duties: (1) Investigate and prosecute on its
own or on complaint by any person, any act or omission of any
public officer or employee, office or agency, when such act or
omission appears to be illegal, unjust, improper, or inefficient. It has
primary jurisdiction over cases cognizable by the Sandiganbayan
and, in the exercise of this primary jurisdiction, it may take over, at
any stage, from any investigatory agency of Government, the
investigations of such cases.
 LGC 61(c): Form and Filing of Administrative Complaints. - A verified
complaint against any erring elective official shall be prepared as
follows: (c) A complaint against any elective barangay official shall
be filed before the sangguniang panlungsod or sangguniang
bayan concerned whose decision shall be final and executory.
 Primary jurisdiction of Ombudsman applies only in cases cognizable
by the Sandiganbayan.
 In cases cognizable by the regular courts, Ombudsman’s jurisdiction
is concurrent with other investigative agencies.
 RA 8749 limits cases cognizable by the Sandiganbayan to public
officials with positions salary grade 27 and higher. Punong barangay
is salary grade 14, so no jurisdiction.
 From the applicable laws, it is clear that the Ombudsman has
concurrent jurisdiction with the sangguniang bayan over
administrative cases against elective barangay officials with salary
grade below 27, such as Rodriguez.
 In administrative cases involving concurrent jurisdiction of 2 or more
disciplining authorities, the body in w/c the complaint is filed first, and
which opts to take cognizance of the case, acquires jurisdiction to the
exclusion of other tribunals exercising concurrent jurisdiction.
 CAB: Since the complaint was filed first in the Ombudsman, and it
opted to assume jurisdiction over the complaint, the Ombudsman’s
exercise of jurisdiction is to the exclusion of the sangguniang bayan.
 Jurisdiction is a matter of law. Once acquired, it is not lost upon the
instance of the parties but continues until the case is terminated.
CAB: When complainants filed their case before the Ombudsman,
jurisdiction was already vested. Jurisdiction could no longer be
transferred to the sangguniang bayan by virtue of a subsequent
complaint filed by the same complainants.
 Under LGC 60, the sangguniang bayan has no power to remove an
elective barangay official. Apart from the Ombudsman, only a proper
court may do so. Unlike the sangguniang bayan, the powers of the
Ombudsman are not merely recommendatory. The Ombudsman is
clothed with authority to directly remove an erring public official other
than members of Congress and the Judiciary who may be removed
only by impeachment.

DISPOSITION: Petition granted. Ombudsman decision upheld.

G. R. No. 159508
BERSAMIN, J.:
The petitioner has directly come to the Court via petition for certiorari[1] filed on
September 4, 2003 to assail the orders dated March 24, 2003 (reversing an earlier
order issued on February 18, 2003 granting his motion to dismiss on the ground of
the action being already barred by prescription, and reinstating the action),[2]
April 21, 2003 (denying his motion for reconsideration),[3] and August 19, 2003
(denying his second motion for reconsideration and ordering him to file his answer
within 10 days from notice despite the principal defendant not having been yet
validly served with summons and copy of the complaint),[4] all issued by the
Regional Trial Court (RTC), Branch 12, in Malolos City in Civil Case No. 722-M-
2002,[5] an action for the recovery of ownership and possession. He alleges that
respondent Presiding Judge thereby acted with grave abuse of discretion
amounting to lack or excess of jurisdiction.
Antecedents

The present controversy started almost four decades ago when Leodegario B.
Ramos (Ramos), one of the defendants in Civil Case No. 722M- 2002, discovered
that a parcel of land with an area of 1,233 square meters, more or less, which was
a portion of a bigger tract of land with an area of 3,054 square meters, more or
less, located in Meycauayan, Bulacan that he had adjudicated solely to himself
upon his mother's death on November 16, 1982 had been earlier transferred by his
mother to one Ricardo Asuncion, who had, in turn, sold it to the late Rodrigo
Gomez.

On February 1, 1990, Ramos, alleging that Gomez had induced him to sell the
1,233 square meters to Gomez on the understanding that Gomez would settle
Ramos' obligation to three other persons, commenced in the RTC in Valenzuela an
action against Gomez, also known as Domingo Ng Lim, seeking the rescission of
their contract of sale and the payment of damages, docketed as Civil Case No.
3287-V-90 entitled Leodegario B. Ramos v. Rodrigo Gomez, a.k.a. Domingo Ng
Lim.[6]

On October 9, 1990, before the Valenzuela RTC could decide Civil Case No.
3287-V-90 on the merits, Ramos and Gomez entered into a compromise
agreement.[7] The RTC approved their compromise agreement through its
decision rendered on the same date.[8]

The petitioner, being then the counsel of Ramos in Civil Case No. 3287-V-90,
assisted Ramos in entering into the compromise agreement "to finally terminate
this case." The terms and conditions of the compromise agreement were as
follows:
COME NOW, the Parties, assisted by their respective counsels, and
before this Honorable Court, most respectfully submit this
COMPROMISE AGREEMENT for approval, as to finally terminate this
case, the terms and conditions of which being as follows:

1. That out of the total area of Three Thousand and Fifty Four (3,054) sq.
m., more or less, covered by formerly O.C.T. No. P-2492 (M), Registry
of Deeds of Bulacan, known as Lot No. 6821, Cad-337 Lot 4020E, Csd-
04-001618-D, and now by the Reconstituted Transfer Certificate of Title
No. T-10179-P (M) defendant shall cause survey of said property, at its
own expense, to segregate the area of One Thousand Two Hundred
Thirty-Three, (1,233) sq. m. more or less, to take along lines two (2) to
three (3), then to four (4) and up to five (5) of said plan, Csd-04-
001618D;

2. That upon completion of the technical survey and plan, defendant shall
cause the registration of the Deed of Absolute Sale executed by plaintiff
over the 1,233 sq. m. in his favor and that defendant shall deliver the
survey and plan pertaining to the 1,821 sq, m. to the plaintiff with both
parties defraying the cost of registration and titling over their respective
shares;

3. That to carry out the foregoing, plaintiff shall entrust the Owner's
Duplicate of said TCT No. T-10179-P (M), Registry of Deeds of
Meycauayan, Bulacan, to the defendant, upon approval of this
COMPROMISE AGREEMENT by the Court;

4. That upon the approval of this Compromise Agreement plaintiff shall


execute a Deed of Absolute Sale in favor of defendant over the 1,233 sq.
m. surveyed and segregated from the 1,821 sq. m. which should remain
with the plaintiff and to be titled in his name;

5. That plaintiff obligates himself to return his loan obligation to the


defendant, in the principal sum of P 80,000.00 plus P 20,000.00 for the
use thereof, and an additional sum of P 10,000.00 in the concept of
attorney's fees, which sums shall be guaranteed by a post-dated check, in
the amount of P 110,000.00 in plaintiff's name with his prior
endorsement, drawn and issued by plaintiff's counsel, for a period of
Sixty (60) days from October 9, 1990;

6. That in the event the check issued pursuant to paragraph 5 hereof, is


dishonored for any reason whatsoever, upon presentment for payment,
then this Compromise Agreement, shall be considered null and void and
of no effect whatsoever;
7. That upon faithful compliance with the terms and conditions of this
COMPROMISE AGREEMENT and the Decision based thereon, the
parties hereto shall have respectively waived, conceded and abandoned
all claims and rights of action of whatever kind or nature, against each
other over the subject property.

WHEREFORE, premises considered, the parties hereto hereby jointly


and severally pray before this Honorable Court to approve this
COMPROMISE AGREEMENT and thereupon render its Decision based
thereon terminating the case.

One of the stipulations of the compromise agreement was for Ramos to execute a
deed of absolute sale in favor of Gomez respecting the parcel of land with an area
of 1,233 square meters, and covered by Transfer Certificate of Title (TCT) No. T-
13005 P(M) in the name of Ramos.[9] Another stipulation was for the petitioner to
issue post-dated checks totaling P110,000.00 to guarantee the payment by Ramos
of his monetary obligations towards Gomez as stated in the compromise
agreement broken down as follows: (a) P80,000.00 as Ramos' loan obligation to
Gomez; (b) P20,000.00 for the use of the loan; and (c) P10,000.00 as attorney's
fees. Of these amounts, only P80,000.00 was ultimately paid to Gomez, because
the petitioner's check dated April 23, 1991 for the balance of P30,000.00 was
dishonored for insufficiency of funds.

Gomez meanwhile died on November 7, 1990. He was survived by his wife Tsui
Yuk Ying and their minor children (collectively to be referred to as the Estate of
Gomez). The Estate of Gomez sued Ramos and the petitioner for specific
performance in the RTC in Caloocan City to recover the balance of P30,000.00
(Civil Case No. C-15750). On February 28, 1994, however, Civil Case No. C-
15750 was amicably settled through a compromise agreement, whereby the
petitioner directly bound himself to pay to the Estate of Gomez P10,000.00 on or
before March 15, 1994; P10,000.00 on or before April 15, 1994; and P10,000.00
on or before May 15, 1994.
The Estate of Gomez performed the obligations of Gomez under the first
paragraph of the compromise agreement of October 9, 1990 by causing the survey
of the bigger tract of land containing an area of 3,054 square meters, more or less,
in order to segregate the area of 1,233 square meters that should be transferred by
Ramos to Gomez in accordance with Ramos' undertaking under the second
paragraph of the compromise agreement of October 9, 1990. But Ramos failed to
cause the registration of the deed of absolute sale pursuant to the second paragraph
of the compromise agreement of October 9, 1990 despite the Estate of Gomez
having already complied with Gomez's undertaking to deliver the approved survey
plan and to shoulder the expenses for that purpose. Nor did Ramos deliver to the
Estate of Gomez the owner's duplicate copy of TCT No. T-10179 P(M) of the
Registry of Deeds of Meycauayan, Bulacan, as stipulated under the third
paragraph of the compromise agreement of October 9, 1990. Instead, Ramos and
the petitioner caused to be registered the 1,233 square meter portion in Ramos's
name under TCT No. T-13005-P(M) of the Registry of Deeds of Meycauayan,
Bulacan.

Accordingly, on July 6, 1995, the Estate of Gomez brought a complaint for


specific performance against Ramos and the petitioner in the RTC in Valenzuela
(Civil Case No. 4679-V-95)[10] in order to recover the 1,233 square meter lot.
However, the Valenzuela RTC dismissed the complaint on April 1, 1996 upon the
motion of Ramos and the petitioner on the ground of improper venue because the
objective was to recover the ownership and possession of realty situated in
Meycauayan, Bulacan, and because the proper recourse was to enforce the
judgment by compromise Agreement rendered on October 9, 1990 through a
motion for execution.

The Estate of Gomez appealed the order of dismissal to the Court of Appeals
(CA), which ruled on July 24, 2001 to affirm the Valenzuela RTC and to dismiss
the appeal (CA-G.R. CV No. 54231).
On September 20, 2002, the Estate of Gomez commenced Civil Case No. 722-M-
2002 in the Valenzuela RTC, ostensibly to revive the judgment by compromise
rendered on October 9, 1990 in Civil Case No. 3287-V-90, praying that Ramos be
ordered to execute the deed of absolute sale covering the 1,233 square meter lot
pursuant to the fourth stipulation of the compromise agreement of October 9,
1990. The petitioner was impleaded as a party-defendant because of his having
guaranteed the performance by Ramos of his obligation and for having actively
participated in the transaction.

On January 8, 2003, the petitioner moved for the dismissal of Civil Case No. 722-
M-2002, alleging that the action was already barred by res judicata and by
prescription; that he was not a real party-in-interest; and that the amount he had
guaranteed with his personal check had already been paid by Ramos with his own
money.[11]

Initially, on February 18, 2003,[12] the RTC granted the petitioner's motion to
dismiss, finding that the right of action had already prescribed due to more than 12
years having elapsed from the approval of the compromise agreement on October
9, 1990, citing Article 1143 (3) of the Civil Code (which provides a 10-year period
within which a right of action based upon a judgment must be brought from).

On March 24, 2003,[13] however, the RTC reversed itself upon motion of the
Estate of Gomez and set aside its order of February 18, 2003. The RTC reinstated
Civil Case No. 722-M-2002, holding that the filing of the complaint for specific
performance on July 6, 1995 in the Valenzuela RTC (Civil Case No. 4679-V-95)
had interrupted the prescriptive period pursuant to Article 1155 of the Civil Code.

The petitioner sought reconsideration, but the RTC denied his motion for that
purpose on April 21, 2003.
On May 12, 2003, the petitioner filed a second motion for reconsideration,
maintaining that the Estate of Gomez's right of action had already prescribed; and
that the judgment by compromise of October 9, 1990 had already settled the entire
controversy between the parties.

On August 19, 2003,[14] the RTC denied the second motion for reconsideration
for lack of merit.

Hence, this special civil action for certiorari commenced on September 4, 2003
directly in this Court.
Issues

The petitioner insists that:


xxx the lower court acted with grave abuse of discretion, amounting to
lack of, or in excess of jurisdiction, when, after having correctly ordered
the dismissal of the case below, on the ground of prescription under Art.
1144, par. 3, of the Civil Code, it reconsidered and set aside the same, on
the factually baseless and legally untenable Motion for Reconsideration
of Private Respondent, insisting, with grave abuse of discretion, if not
bordering on ignorance of law, and too afraid to face reality, that it is
Art. 1155 of the same code, as invoked by Private Respondents, that
applies, and required herein petitioner to file his answer, despite
petitioner's first Motion for Reconsideration, which it treated as a mere
scrap of paper, yet, at the same [sic] again it insisted that Article 1155 of
the Civil Code should apply, and, thereafter when, with like, if not
greater grave abuse of discretion, amounting to lack, or in excess of
jurisdiction, it again denied petitioner's Second Motion for
Reconsideration for lack of merit, and giving petitioner a non-extendible
period of ten [10] days from notice, to file his answer.[15]

In his reply to the Estate of Gomez's comment,[16] the petitioner elucidated as


follows:
[1] Whether or not, the Honorable public respondent Judge gravely
abused his discretion, amounting to lack of, or in excess of jurisdiction,
when, after ordered the dismissal of Civil Case No. 722-M-2002, as
prescription has set in, under Art. 1143 of the Civil Code, he set aside
and reconsidered his said Order, on motion of plaintiff, by thereafter
denied petitioner's Motion for Reconsideration, and Second Motion for
Reconsideration, insisting, despite his being presumed to know the law,
that the said action is not barred by prescription, under Art. 1145 of the
Civil Code;

[2] Whether or not, the present pending action, Civil Case No. 722M-
2002, before Branch 12 of the Regional Trial Court of Malolos, Bulacan,
is barred, and should be ordered be dismissed, on the ground of
prescription, under the law and the rules, and applicable jurisprudence.

[3] Whether or not, the same action may be dismissed on other valid
grounds.[17]

The petitioner submits that Civil Case No. 722-M-2002 was one for the revival of
the judgment upon a compromise agreement rendered in Civil Case No. 3287-V-
90 that attained finality on October 9, 1990; that considering that an action for
revival must be filed within 10 years from the date of finality, pursuant to Article
1144 of the Civil Code,[18] in relation to Section 6, Rule 39 of the Rules of
Court,[19] Civil Case No. 722-M-2002 was already barred by prescription, having
been filed beyond the 10-year prescriptive period; that the RTC gravely abused its
discretion in reinstating the complaint despite prescription having already set in;
that the dismissal of Civil Case No. 722-M-2002 was proper also because the
judgment had already been fully satisfied; that the claim relative to the 1,233
square meter lot under the compromise agreement had been waived, abandoned, or
otherwise extinguished on account of the failure of the Estate of Gomez's counsel
to move for the issuance of a writ of execution; and that the Estate of Gomez could
not rely upon the pendency and effects of the appeal from the action for specific
performance after its dismissal had been affirmed by the CA on grounds of
improper venue, the plaintiff's lack of personality, and improper remedy (due to
the proper remedy being by execution of the judgment).

The Estate of Gomez countered that the filing on July 6, 1995 of the action for
specific performance in the RTC in Valenzuela stopped the running of the
prescriptive period; that the period commenced to run again after the CA
dismissed that action on July 24, 2001; that the total elapsed period was only five
years and 11 months; and that the action for the revival of judgment filed on
September 20, 2002 was within the period of 10 years to enforce a final and
executory judgment by action.
Ruling

We dismiss the petition for certiorari.

The orders that the petitioner seeks to challenge and to annul are the orders
denying his motion to dismiss. It is settled, however, that an order denying a
motion to dismiss, being merely interlocutory, cannot be the basis of a petition for
certiorari. An interlocutory order is not the proper subject of a certiorari
challenge by virtue of its not terminating the proceedings in which it is issued. To
allow such order to be the subject of review by certiorari not only delays the
administration of justice, but also unduly burdens the courts.[20]

But a petition for certiorari may be filed to assail an interlocutory order if it is


issued without jurisdiction, or with excess of jurisdiction, or in grave abuse of
discretion amounting to lack or excess of jurisdiction. This is because as to such
order there is no appeal, or any plain, speedy, and adequate remedy in the ordinary
course of law. Rule 65 of the Rules of Court expressly recognizes the exception by
providing as follows:
Section 1. Petition for certiorari. When any tribunal, board or officer
exercising judicial or quasi-judicial functions has acted without or in
excess of its or his jurisdiction, or with grave abuse of discretion
amounting to lack or excess of jurisdiction, and there is no appeal, or any
plain, speedy, and adequate remedy in the ordinary course of law, a
person aggrieved thereby may file a verified petition in the proper court,
alleging the facts with certainty and praying that judgment be rendered
annulling or modifying the proceedings of such tribunal, board or officer,
and granting such incidental reliefs as law and justice may require.

The petition shall be accompanied by a certified true copy of the


judgment, order or resolution subject thereof, copies of all pleadings and
documents relevant and pertinent thereto, and a sworn certification of
non- forum shopping as provided in the third paragraph of section 3,
Rule 46. (1a)

The exception does not apply to this challenge. The petitioner has not
demonstrated how the assailed orders could have been issued without jurisdiction,
or with excess of jurisdiction, or in grave abuse of discretion amounting to lack or
excess of jurisdiction. Nor has he convinced us that he had no plain, speedy, and
adequate remedy in the ordinary course of law. In fact and in law, he has, like
filing his answer and going to pre-trial and trial. In the end, should he still have the
need to seek the review of the decision of the RTC, he could also even appeal the
denial of the motion to dismiss. That, in reality, was his proper remedy in the
ordinary course of law.

Yet another reason to dismiss the petition for certiorari exists. Although the Court,
the CA and the RTC have concurrence of jurisdiction to issue writs of certiorari,
the petitioner had no unrestrained freedom to choose which among the several
courts might his petition for certiorari be filed in. In other words, he must observe
the hierarchy of courts, the policy in relation to which has been explicitly defined
in Section 4 of Rule 65 concerning the petitions for the extraordinary writs of
certiorari, prohibition and mandamus, to wit:
Section 4. When and where petition filed. - The petition shall be filed not
later than sixty (60) days from notice of the judgment, order or
resolution. In case a motion for reconsideration or new trial is timely
filed, whether such motion is required or not, the sixty (60) day period
shall be counted from notice of the denial of the said motion.

The petition shall be filed in the Supreme Court or, if it relates to the
acts or omissions of a lower court or of a corporation, board, officer
or person, in the Regional Trial Court exercising jurisdiction over
the territorial area as defined by the Supreme Court. It may also be
filed in the Court of Appeals whether or not the same is in the aid of
its appellate jurisdiction, or in the Sandiganbayan if it is in aid of its
appellate jurisdiction. If it involves the acts or omissions of a quasi-
judicial agency, unless otherwise provided by law or these rules, the
petition shall be filed in and cognizable only by the Court of Appeals.

No extension of time to file the petition shall be granted except for


compelling reason and in no case exceeding fifteen (15) days. (4a)[21]
(Emphasis supplied)

Accordingly, his direct filing of the petition for certiorari in this Court instead of
in the CA should be disallowed considering that he did not present in the petition
any special and compelling reasons to support his choice of this Court as the
forum.

The Court must enjoin the observance of the policy on the hierarchy of courts, and
now affirms that the policy is not to be ignored without serious consequences. The
strictness of the policy is designed to shield the Court from having to deal with
causes that are also well within the competence of the lower courts, and thus leave
time to the Court to deal with the more fundamental and more essential tasks that
the Constitution has assigned to it. The Court may act on petitions for the
extraordinary writs of certiorari, prohibition and mandamus only when absolutely
necessary or when serious and important reasons exist to justify an exception to
the policy. This was why the Court stressed in Vergara, Sr. v. Suelto:[22]
xxx. The Supreme Court is a court of last resort, and must so remain if it
is to satisfactorily perform the functions assigned to it by the
fundamental charter and immemorial tradition. It cannot and should not
be burdened with the task of dealing with causes in the first instance. Its
original jurisdiction to issue the so-called extraordinary writs should be
exercised only where absolutely necessary or where serious and
important reasons exist therefor. Hence, that jurisdiction should generally
be exercised relative to actions or proceedings before the Court of
Appeals, or before constitutional or other tribunals, bodies or agencies
whose acts for some reason or another are not controllable by the Court
of Appeals. Where the issuance of an extraordinary writ is also within
the competence of the Court of Appeals or a Regional Trial Court, it is in
either of these courts that the specific action for the writ's procurement
must be presented. This is and should continue to be the policy in this
regard, a policy that courts and lawyers must strictly observe. (Emphasis
supplied)

In People v. Cuaresma,[23] the Court has also amplified the need for strict
adherence to the policy of hierarchy of courts. There, noting "a growing tendency
on the part of litigants and lawyers to have their applications for the so-called
extraordinary writs, and sometimes even their appeals, passed upon and
adjudicated directly and immediately by the highest tribunal of the land," the
Court has cautioned lawyers and litigants against taking a direct resort to the
highest tribunal, viz:
xxx. This Court's original jurisdiction to issue writs of certiorari (as
well as prohibition, mandamus, quo warranto, habeas corpus and
injunction) is not exclusive. It is shared by this Court with Regional
Trial Courts x x x, which may issue the writ, enforceable in any part of
their respective regions. It is also shared by this Court, and by the
Regional Trial Court, with the Court of Appeals x x x, although prior to
the effectivity of Batas Pambansa Bilang 129 on August 14, 1981, the
latter's competence to issue the extraordinary writs was restricted to those
"in aid of its appellate jurisdiction." This concurrence of jurisdiction is
not, however, to be taken as according to parties seeking any of the
writs an absolute, unrestrained freedom of choice of the court to
which application therefor will be directed. There is after all a
hierarchy of courts. That hierarchy is determinative of the venue of
appeals, and should also serve as a general determinant of the appropriate
forum for petitions for the extraordinary writs. A becoming regard for
that judicial hierarchy most certainly indicates that petitions for the
issuance of extraordinary writs against first level ("inferior") courts
should be filed with the Regional Trial Court, and those against the
latter, with the Court of Appeals. A direct invocation of the Supreme
Court's original jurisdiction to issue these writs should be allowed
only when there are special and important reasons therefor, clearly
and specifically set out in the petition. This is established policy. It is
a policy that is necessary to prevent inordinate demands upon the
Court's time and attention which are better devoted to those matters
within its exclusive jurisdiction, and to prevent further over-
crowding of the Court's docket. Indeed, the removal of the restriction
on the jurisdiction of the Court of Appeals in this regard, supra resulting
from the deletion of the qualifying phrase, "in aid of its appellate
jurisdiction" was evidently intended precisely to relieve this Court pro
tanto of the burden of dealing with applications for the extraordinary
writs which, but for the expansion of the Appellate Court corresponding
jurisdiction, would have had to be filed with it.

xxxx

The Court therefore closes this decision with the declaration for the
information and evidence of all concerned, that it will not only
continue to enforce the policy, but will require a more strict
observance thereof. (Emphasis supplied)

There being no special, important or compelling reason that justified the direct
filing of the petition for certiorari in this Court in violation of the policy on
hierarchy of courts, its outright dismissal is unavoidable.

Still, even granting that the petition for certiorari might be directly filed in this
Court, its dismissal must also follow because its consideration and resolution
would unavoidably demand the consideration and evaluation of evidentiary
matters. The Court is not a trier of facts, and cannot accept the petition for
certiorari for that reason.

Although commenced ostensibly for the recovery of possession and ownership of


real property, Civil Case No. 722-M-2002 was really an action to revive the
judgment by compromise dated October 9, 1990 because the ultimate outcome
would be no other than to order the execution of the judgment by compromise.
Indeed, it has been held that "there is no substantial difference between an action
expressly called one for revival of judgment and an action for recovery of property
under a right adjudged under and evidenced by a final judgment."[24] In addition,
the parties themselves have treated the complaint in Civil Case No. 722-M-2002
as one for revival. Accordingly, the parties should be fully heard on their
respective claims like in any other independent action.

The petitioner's defense of prescription to bar Civil Case No. 722-M2002 presents
another evidentiary concern. Article 1144 of the Civil Code requires, indeed, that
an action to revive a judgment must be brought before it is barred by prescription,
which was ten years from the accrual of the right of action.[25] It is clear,
however, that such a defense could not be determined in the hearing of the
petitioner's motion to dismiss considering that the complaint did not show on its
face that the period to bring the action to revive had already lapsed. An allegation
of prescription, as the Court put it in Pineda v. Heirs of Eliseo Guevara,[26] "can
effectively be used in a motion to dismiss only when the complaint on its face
shows that indeed the action has already prescribed, [o]therwise, the issue of
prescription is one involving evidentiary matters requiring a full blown trial on the
merits and cannot be determined in a mere motion to dismiss."

At any rate, the mere lapse of the period per se did not render the judgment stale
within the context of the law on prescription, for events that effectively suspended
the running of the period of limitation might have intervened. In other words, the
Estate of Gomez was not precluded from showing such events, if any. The Court
recognized this possibility of suspension in Lancita v. Magbanua:[27]

In computing the time limited for suing out of an execution, although there is
authority to the contrary, the general rule is that there should not be included the
time when execution is stayed, either by agreement of the parties for a definite
time, by injunction, by the taking of an appeal or writ of error so as to operate as a
supersedeas, by the death of a party or otherwise. Any interruption or delay
occasioned by the debtor will extend the time within which the writ may be issued
without scire facias.

Verily, the need to prove the existence or non-existence of significant matters, like
supervening events, in order to show either that Civil Case No. 722-M-2002 was
batTed by prescription or not was present and undeniable. Moreover, the petitioner
himself raised factual issues in his motion to dismiss, like his averment of full
payment or discharge of the obligation of Ramos and the waiver or abandonment
of rights under the compromise agreement. The proof thereon cannot be received
in certiorari proceedings before the Court, but should be established in the RTC.

WHEREFORE, the Court DISMISSES the petition for certiorari; and


DIRECTS the petitioner to pay the cost of suit.

SO ORDERED.

Sereno, C.J., Leonardo-De Castro, Villarama, Jr., and Reyes, JJ., concur.
ERNESTO DY v. GINA M. BIBAT-PALAMOS +

G.R. No. 196200


MENDOZA, J.:
This petition for certiorari under Rule 65 of the 1997 Revised Rules of Civil
Procedure questions the December 13, 2010 and March 7, 2011 Orders[1] of the
Regional Trial Court of Makati, Branch 64 (RTC), in Civil Case No. 92-2311,
granting the motion for execution of petitioner, but denying his prayer for the
return of his cargo vessel in the condition when the possession thereof was seized
from him.
The Facts

The present controversy finds its roots in the Court's decision in Orix Metro
Leasing and Finance Corporation v. M/V "Pilar-I" and Spouses Ernesto Dy and
Lourdes Dy[2] involving the same parties. The facts, as culled from the Court's
decision in the said case and the records, are not disputed by the parties.

Petitioner Ernesto Dy (petitioner) and his wife, Lourdes Dy (Lourdes), were the
proprietors of Limchia Enterprises which was engaged in the shipping business.
In 1990, Limchia Enterprises, with Lourdes as co-maker, obtained a loan from
Orix Metro Leasing and Finance Corporation (respondent) to fund its acquisition
of M/V Pilar-I, a cargo vessel. As additional security for the loan, Limchia
Enterprises executed the Deed of Chattel Mortgage over M/V Pilar-I.[3]

Due to financial losses suffered when M/V Pilar-I was attacked by pirates,
Spouses Dy failed to make the scheduled payments as required in their promissory
note. After receiving several demand letters from respondent, Spouses Dy applied
for the restructuring of their loan. Meanwhile, Lourdes issued several checks to
cover the remainder of their loan but the same were dishonored by the bank,
prompting respondent to institute a criminal complaint for violation of the
Bouncing Checks Law. Lourdes appealed to respondent with a new proposal to
update their outstanding loan obligations.[4]

On August 18, 1992, respondent filed the Complaint and Petition for Extrajudicial
Foreclosure of Preferred Ship Mortgage under Presidential Decree No. 1521 with
Urgent Prayer for Attachment with the RTC. Following the filing of an affidavit of
merit and the posting of bond by respondent, the RTC ordered the seizure of M/V
Pilar-I and turned over its possession to respondent. On September 28, 1994,
respondent transferred all of its rights, title to and interests, as mortgagee, in M/V
Pilar-I to Colorado Shipyard Corporation (Colorado).[5]

On July 31, 1997, the RTC rendered a decision in favor of Spouses Dy, ruling that
they had not yet defaulted on their loan because respondent agreed to a
restructured schedule of payment. There being no default, the foreclosure of the
chattel mortgage on M/V Pilar-I was premature. The RTC ordered that the vessel
be returned to Spouses Dy.[6] This was affirmed by the Court of Appeals (CA),
with the modification that Spouses Dy be ordered to reimburse the respondent for
repair and drydocking expenses while the vessel was in the latter's possession.[7]
On appeal, the Court promulgated its Decision, dated September 11, 2009,
upholding the findings of the CA but deleting the order requiring Spouses Dy to
reimburse respondent.[8]

Consequently, on August 17, 2010, petitioner filed a motion for execution of


judgment with the RTC. In the intervening period, Colorado filed its
Manifestation/Motion, dated July 29, 2010, informing the RTC that M/V Pilar-I,
which was in its possession, had sustained severe damage and deterioration and
had sunk in its shipyard because of its exposure to the elements. For this reason, it
sought permission from the court to cut the sunken vessel into pieces, sell its parts
and deposit the proceeds in escrow.[9] In his Comment/Objection, petitioner
insisted that he had the right to require that the vessel be returned to him in the
same condition that it had been at the time it was wrongfully seized by respondent
or, should it no longer be possible, that another vessel of the same tonnage, length
and beam similar to that of M/V Pilar-I be delivered.[10] Colorado, however,
responded that the vessel had suffered severe damage and deterioration that
refloating or restoring it to its former condition would be futile, impossible and
very costly; and should petitioner persist in his demand that the ship be refloated,
it should be done at the expense of the party adjudged by the court to pay the
same.[11]

The RTC issued its questioned December 13, 2010 Order granting the motion for
execution but denying petitioner's prayer for the return of M/V Pilar-I in the same
state in which it was taken by respondent. In so resolving, the RTC ratiocinated:
First, the judgment of the Supreme Court does not require the delivery of
M/V Pilar in the state the defendants wanted it to be. Secondly, said
judgment has now become final and it is axiomatic that after judgment
has become executory, the court cannot amend the same, except: x x x
None of the three circumstances where a final and executory judgment
may be amended is present in this case. And third, the present
deplorable state of M/V Pilar certainly did not happen overnight, thus,
defendants should have brought it to the attention of this Court, the Court
of Appeals or the Supreme Court after it became apparent. Their
inaction until after the judgment has become final, executory and
immutable rendered whatever right they may have to remedy the
situation to be nugatory. [Underlining supplied]

Petitioner moved for reconsideration but the motion was denied by the RTC in its
March 7, 2011 Order.[12]

Hence, this petition.


The Issues

Petitioner raises the following issues in its Memorandum:


1. Whether or not the rule on hierarchy of courts is applicable to the
instant petition?

2. Whether or not the honorable trial court gravely abused its


discretion, amounting to lack or excess of jurisdiction, in finding that
petitioner is not entitled to the return of M/V Pilar-1 in the condition
that it had when it was wrongfully seized by Orix Metro, or in the
alternative, to a vessel of similar tonnage, length, beam, and other
particulars as M/V Pilar-1;

3. Whether or not petitioner is estopped from asking for the return


of the vessel in the condition it had at the time it was seized?

4. Whether or not it was petitioner's duty to look out for the vessel's
condition?[13]

To be succinct, only two central issues need to be resolved: (1) whether petitioner
was justified in resorting directly to this Court via a petition for certiorari under
Rule 65; and (2) whether petitioner is entitled to the return of M/V Pilar-I in the
same condition when it was seized by respondent.
The Court's Ruling

The Court finds the petition to be partly meritorious.

Hierarchy of Courts; Direct Resort


To The Supreme Court Justified

Petitioner argues that his situation calls for the direct invocation of this Court's
jurisdiction in the interest of justice. Moreover, as pointed out by the RTC, what
is involved is a judgment of the Court which the lower courts cannot modify.
Hence, petitioner deemed it proper to bring this case immediately to the attention
of this Court. Lastly, petitioner claims that the present case involves a novel issue
of law that is, whether in an action to recover, a defendant in wrongful possession
of the subject matter in litigation may be allowed to return the same in a
deteriorated condition without any liability.[14]

Respondent, on the other hand, contends that the petition should have been filed
with the CA, following the doctrine of hierarchy of courts. It pointed out that
petitioner failed to state any special or important reason or any exceptional and
compelling circumstance which would warrant a direct recourse to this Court.[15]
Under the principle of hierarchy of courts, direct recourse to this Court is improper
because the Supreme Court is a court of last resort and must remain to be so in
order for it to satisfactorily perform its constitutional functions, thereby allowing it
to devote its time and attention to matters within its exclusive jurisdiction and
preventing the overcrowding of its docket.[16] Nonetheless, the invocation of this
Court's original jurisdiction to issue writs of certiorari has been allowed in certain
instances on the ground of special and important reasons clearly stated in the
petition, such as, (1) when dictated by the public welfare and the advancement of
public policy; (2) when demanded by the broader interest of justice; (3) when the
challenged orders were patent nullities; or (4) when analogous exceptional and
compelling circumstances called for and justified the immediate and direct
handling of the case.[17]

This case falls under one of the exceptions to the principle of hierarchy of courts.
Justice demands that this Court take cognizance of this case to put an end to the
controversy and resolve the matter which has been dragging on for more than
twenty (20) years. Moreover, in light of the fact that what is involved is a final
judgment promulgated by this Court, it is but proper for petitioner to call upon its
original jurisdiction and seek final clarification.

Wrong Mode of Appeal;


Exception

Petitioner asserts that the RTC committed grave abuse of discretion when it failed
to rule in his favor despite the fact that he had been deprived by respondent of his
property rights over M/V Pilar-I for the past eighteen (18) years. Moreover, the
change in the situation of the parties calls for a relaxation of the rules which would
make the execution of the earlier decision of this Court inequitable or unjust.
According to petitioner, for the RTC to allow respondent to return the ship to him
in its severely damaged and deteriorated condition without any liability would be
to reward bad faith.[18]

Conversely, respondent submits that there was no grave abuse of discretion on the
part of the RTC as the latter merely observed due process and followed the
principle that an execution order may not vary or go beyond the terms of the
judgment it seeks to enforce.[19] Respondent adds that the proper remedy should
have been an ordinary appeal, where a factual review of the records can be made
to determine the condition of the ship at the time it was taken from petitioner, and
not a special civil action for certiorari.[20]

There are considerable differences between an ordinary appeal and a petition for
certiorari which have been exhaustively discussed by this Court in countless
cases. The remedy for errors of judgment, whether based on the law or the facts of
the case or on the wisdom or legal soundness of a decision, is an ordinary
appeal.[21] In contrast, a petition for certiorari under Rule 65 is an original action
designed to correct errors of jurisdiction, defined to be those "in which the act
complained of was issued by the court, officer, or quasi-judicial body without or in
excess of jurisdiction, or with grave abuse of discretion which is tantamount to
lack of in excess of jurisdiction."[22] A court or tribunal can only be considered
to have acted with grave abuse of discretion if its exercise of judgment was so
whimsical and capricious as to be equivalent to a lack of jurisdiction. The abuse
must be extremely patent and gross that it would amount to an "evasion of a
positive duty or to virtual refusal to perform a duty enjoined by law, or to act at all
in contemplation of law, as where the power is exercised in an arbitrary and
despotic manner by reason of passion and hostility."[23]

Therefore, a misappreciation of evidence on the part of the lower court, as asserted


by petitioner, may only be reviewed by appeal and not by certiorari because the
issue raised by the petitioner does not involve any jurisdictional ground.[24] It is
a general rule of procedural law that when a party adopts an inappropriate mode of
appeal, his petition may be dismissed outright to prevent the erring party from
benefiting from his neglect and mistakes.[25] There are exceptions to this
otherwise ironclad rule, however. One is when the strict application of procedural
technicalities would hinder the expeditious disposition of this case on the
merits,[26] such as in this case.

Petitioner Not Barred from Demanding


Return of the Vessel in its Former Condition

Petitioner insists that it is respondent who should bear the responsibility for the
deterioration of the vessel because the latter, despite having in its possession the
vessel M/V Pilar-I during the pendency of the foreclosure proceedings, failed to
inform the court and petitioner himself about the actual condition of the ship. For
estoppel to take effect, there must be knowledge of the real facts by the party
sought to be estopped and reliance by the party claiming estoppel on the
representation made by the former. In this case, petitioner cannot be estopped
from asking for the return of the vessel in the condition that it had been at the time
it was seized by respondent because he had not known of the deteriorated
condition of the ship.[27]

On the contrary, respondent argues that petitioner is barred from asking for a
modification of the judgment since he never prayed for the return of M/V Pilar-I in
the same condition that it had been at the time it was seized.[28] Petitioner could
have prayed for such relief in his prior pleadings and presented evidence thereon
before the judgment became final and executory. During the course of the trial,
and even at the appellate phase of the case, petitioner failed to ask the courts to
look into the naturally foreseeable depreciation of M/V Pilar-I and to determine
who should pay for the wear and tear of the vessel. Consequently, petitioner can
no longer pursue such relief for the first time at this very late stage.[29] Moreover,
respondent posits that it can only be held liable for the restoration and replacement
of the vessel if it can be proven that M/V Pilar-I deteriorated through the fault of
respondent. Nowhere in the prior decision of this Court, however, does it appear
that respondent was found to have been negligent in its care of the vessel. In fact,
respondent points out that, for a certain period, it even paid for the repair and
maintenance of the vessel and engaged the services of security guards to watch
over the vessel. It reasons that the vessel's deterioration was necessarily due to its
exposure to sea water and the natural elements for the almost twenty years that it
was docked in the Colorado shipyard.[30]

On this matter, the Court finds for petitioner.

This Court is not unaware of the doctrine of immutability of judgments. When a


judgment becomes final and executory, it is made immutable and unalterable,
meaning it can no longer be modified in any respect either by the court which
rendered it or even by this Court. Its purpose is to avoid delay in the orderly
administration of justice and to put an end to judicial controversies. Even at the
risk of occasional errors, public policy and sound practice dictate that judgments
must become final at some point.[31]

As with every rule, however, this admits of certain exceptions. When a


supervening event renders the execution of a judgment impossible or unjust, the
interested party can petition the court to modify the judgment to harmonize it with
justice and the facts.[32] A supervening event is a fact which transpires or a new
circumstance which develops after a judgment has become final and executory.
This includes matters which the parties were unaware of prior to or during trial
because they were not yet in existence at that time.[33]

In this case, the sinking of M/V Pilar-I can be considered a supervening event.
Petitioner, who did not have possession of the ship, was only informed of its
destruction when Colorado filed its Manifestation, dated July 29, 2010, long after
the September 11, 2009 Decision of this Court in Orix Metro Leasing and Finance
Corporation v. M/V "Pilar-I" and Spouses Ernesto Dy and Lourdes Dy attained
finality on January 19, 2010. During the course of the proceedings in the RTC, the
CA and this Court, petitioner could not have known of the worsened condition of
the vessel because it was in the possession of Colorado.

It could be argued that petitioner and his lawyer should have had the foresight to
ask for the return of the vessel in its former condition at the time respondent took
possession of the same during the proceedings in the earlier case. Nonetheless, the
modification of the Court's decision is warranted by the superseding
circumstances, that is, the severe damage to the vessel subject of the case and the
belated delivery of this information to the courts by the party in possession of the
same.

Having declared that a modification of our earlier judgment is permissible in light


of the exceptional incident present in this case, the Court further rules that
petitioner is entitled to the return of M/V Pilar-I in the same condition in which
respondent took possession of it. Considering, however, that this is no longer
possible, then respondent should pay petitioner the value of the ship at such time.

This disposition is not without precedent. In the case of Metro Manila Transit
Corporation v. D.M. Consortium, Inc.,[34] D.M. Consortium, Inc. (DMCI)
acquired 228 buses under a lease purchase agreement with Metro Manila Transit
Corporation (MMTC). MMTC later alleged that DMCI was in default of its
amortization, as a result of which, MMTC took possession of all the buses. This
Court upheld the right of DMCI, after having been unjustly denied of its right of
possession to several buses, to have them returned by MMTC. Considering,
however, that the buses could no longer be returned in their original state, the
Court sustained the resolution of the CA ordering MMTC to pay DMCI the value
of the buses at the time of repossession.

The aforecited case finds application to the present situation of petitioner. After
having been deprived of his vessel for almost two decades, through no fault of his
own, it would be the height of injustice to permit the return of M/V Pilar-I to
petitioner in pieces, especially after a judgment by this very same Court ordering
respondent to restore possession of the vessel to petitioner. To do so would leave
petitioner with nothing but a hollow and illusory victory for although the Court
ruled in his favor and declared that respondent wrongfully took possession of his
vessel, he could no longer enjoy the beneficial use of his extremely deteriorated
vessel that it is no longer seaworthy and has no other commercial value but for the
sale of its parts as scrap.

Moreover, the incongruity only becomes more palpable when consideration is


taken of the fact that petitioner's obligation to respondent, for which the now
practically worthless vessel serves as security, is still outstanding.[35] The Court
cannot countenance such an absurd outcome. It could not have been the intention
of this Court to perpetrate an injustice in the guise of a favorable decision. As the
court of last resort, this Court is the final bastion of justice where litigants can
hope to correct any error made in the lower courts.

WHEREFORE, the petition is PARTIALLY GRANTED. Respondent is


ordered to pay petitioner the value of M/V Pilar-I at the time it was wrongfully
seized by it. The case is hereby REMANDED to the Regional Trial Court,
Branch 64, Makati City, for the proper determination of the value of the vessel at
said time.

SO ORDERED.

Velasco, Jr., (Chairperson), Peralta, Abad, and Leonen, JJ., concur.

COMMISSION ON ELECTIONS AND THE ELECTION OFFICER OF


BACOLOD CITY, ATTY. MAVIL V. MAJARUCON, Respondents.
G.R. No. 205728 January 21, 2015
PONENTE: Leonen
TOPIC: Right to expression, right to political speech, right to property

FACTS:
On February 21, 2013, petitioners posted two (2) tarpaulins within a
private compound housing the San Sebastian Cathedral of Bacolod. Each tarpaulin
was approximately six feet (6′) by ten feet (10′) in size. They were posted on the
front walls of the cathedral within public view. The first tarpaulin contains the
message “IBASURA RH Law” referring to the Reproductive Health Law of 2012
or Republic Act No. 10354. The second tarpaulin is the subject of the present case.
This tarpaulin contains the heading “Conscience Vote” and lists candidates as
either “(Anti-RH) Team Buhay” with a check mark, or “(Pro-RH) Team Patay”
with an “X” mark. The electoral candidates were classified according to their vote
on the adoption of Republic Act No. 10354, otherwise known as the RH Law.
Those who voted for the passing of the law were classified by petitioners as
comprising “Team Patay,” while those who voted against it form “Team Buhay.”
Respondents conceded that the tarpaulin was neither sponsored nor paid
for by any candidate. Petitioners also conceded that the tarpaulin contains names
ofcandidates for the 2013 elections, but not of politicians who helped in the
passage of the RH Law but were not candidates for that election.
ISSUES:
1. Whether or not the size limitation and its reasonableness of the tarpaulin is a
political question, hence not within the ambit of the Supreme Court’s power
of review.
2. Whether or not the petitioners violated the principle of exhaustion of
administrative remedies as the case was not brought first before the
COMELEC En Banc or any if its divisions.
3. Whether or not COMELEC may regulate expressions made by private
citizens.
4. Whether or not the assailed notice and letter for the removal of the tarpaulin
violated petitioners’ fundamental right to freedom of expression.
5. Whether the order for removal of the tarpaulin is a content-based or content-
neutral regulation.
6. Whether or not there was violation of petitioners’ right to property.
7. Whether or not the tarpaulin and its message are considered religious speech.

HELD:

FIRST ISSUE: No.


The Court ruled that the present case does not call for the exercise of
prudence or modesty. There is no political question. It can be acted upon by this
court through the expanded jurisdiction granted to this court through Article VIII,
Section 1 of the Constitution..
The concept of a political question never precludes judicial review when
the act of a constitutional organ infringes upon a fundamental individual or
collective right. Even assuming arguendo that the COMELEC did have the
discretion to choose the manner of regulation of the tarpaulin in question, it cannot
do so by abridging the fundamental right to expression.
Also the Court said that in our jurisdiction, the determination of whether
an issue involves a truly political and non-justiciable question lies in the answer to
the question of whether there are constitutionally imposed limits on powers or
functions conferred upon political bodies. If there are, then our courts are duty-
bound to examine whether the branch or instrumentality of the government
properly acted within such limits.
A political question will not be considered justiciable if there are no
constitutionally imposed limits on powers or functions conferred upon political
bodies. Hence, the existence of constitutionally imposed limits justifies subjecting
the official actions of the body to the scrutiny and review of this court.
In this case, the Bill of Rights gives the utmost deference to the right to
free speech. Any instance that this right may be abridged demands judicial
scrutiny. It does not fall squarely into any doubt that a political question brings.
SECOND ISSUE: No.
The Court held that the argument on exhaustion of administrative
remedies is not proper in this case.
Despite the alleged non-exhaustion of administrative remedies, it is clear
that the controversy is already ripe for adjudication. Ripeness is the “prerequisite
that something had by then been accomplished or performed by either branch or in
this case, organ of government before a court may come into the picture.”
Petitioners’ exercise of their right to speech, given the message and their
medium, had understandable relevance especially during the elections.
COMELEC’s letter threatening the filing of the election offense against petitioners
is already an actionable infringement of this right. The impending threat of
criminal litigation is enough to curtail petitioners’ speech.
In the context of this case, exhaustion of their administrative remedies as
COMELEC suggested in their pleadings prolongs the violation of their freedom of
speech.
THIRD ISSUE: No.
Respondents cite the Constitution, laws, and jurisprudence to support
their position that they had the power to regulate the tarpaulin. However, the Court
held that all of these provisions pertain to candidates and political parties.
Petitioners are not candidates. Neither do they belong to any political party.
COMELEC does not have the authority to regulate the enjoyment of the preferred
right to freedom of expression exercised by a non-candidate in this case.
FOURTH ISSUE: Yes.
The Court held that every citizen’s expression with political
consequences enjoys a high degree of protection.
Moreover, the respondent’s argument that the tarpaulin is election
propaganda, being petitioners’ way of endorsing candidates who voted against the
RH Law and rejecting those who voted for it, holds no water.
The Court held that while the tarpaulin may influence the success or
failure of the named candidates and political parties, this does not necessarily
mean it is election propaganda. The tarpaulin was not paid for or posted “in return
for consideration” by any candidate, political party, or party-list group.
By interpreting the law, it is clear that personal opinions are not
included, while sponsored messages are covered.
The content of the tarpaulin is a political speech
Political speech refers to speech “both intended and received as a contribution to
public deliberation about some issue,” “fostering informed and civic minded
deliberation.” On the other hand, commercial speech has been defined as speech
that does “no more than propose a commercial transaction.” The expression
resulting from the content of the tarpaulin is, however, definitely political speech.
FIFTH ISSUE: Content-based regulation.
Content-based restraint or censorship refers to restrictions “based on the
subject matter of the utterance or speech.” In contrast, content-neutral regulation
includes controls merely on the incidents of the speech such as time, place, or
manner of the speech.
The Court held that the regulation involved at bar is content-based. The
tarpaulin content is not easily divorced from the size of its medium.
Content-based regulation bears a heavy presumption of invalidity, and
this court has used the clear and present danger rule as measure.
Under this rule, “the evil consequences sought to be prevented must be
substantive, ‘extremely serious and the degree of imminence extremely high.’”
“Only when the challenged act has overcome the clear and present danger rule will
it pass constitutional muster, with the government having the burden of
overcoming the presumed unconstitutionality.”
Even with the clear and present danger test, respondents failed to justify
the regulation. There is no compelling and substantial state interest endangered by
the posting of the tarpaulin as to justify curtailment of the right of freedom of
expression. There is no reason for the state to minimize the right of non-candidate
petitioners to post the tarpaulin in their private property. The size of the tarpaulin
does not affect anyone else’s constitutional rights.
SIXTH ISSUE: Yes.
The Court held that even though the tarpaulin is readily seen by the
public, the tarpaulin remains the private property of petitioners. Their right to use
their property is likewise protected by the Constitution.
Any regulation, therefore, which operates as an effective confiscation of
private property or constitutes an arbitrary or unreasonable infringement of
property rights is void, because it is repugnant to the constitutional guaranties of
due process and equal protection of the laws.
The Court in Adiong case held that a restriction that regulates where
decals and stickers should be posted is “so broad that it encompasses even the
citizen’s private property.” Consequently, it violates Article III, Section 1 of the
Constitution which provides that no person shall be deprived of his property
without due process of law.
SEVENTH ISSUE: No.
The Court held that the church doctrines relied upon by petitioners are
not binding upon this court. The position of the Catholic religion in the Philippines
as regards the RH Law does not suffice to qualify the posting by one of its
members of a tarpaulin as religious speech solely on such basis. The enumeration
of candidates on the face of the tarpaulin precludes any doubt as to its nature as
speech with political consequences and not religious speech.
Doctrine of benevolent neutrality
With religion looked upon with benevolence and not hostility,
benevolent neutrality allows accommodation of religion under certain
circumstances. Accommodations are government policies that take religion
specifically into account not to promote the government’s favored form of
religion, but to allow individuals and groups to exercise their religion without
hindrance. Their purpose or effect therefore is to remove a burden on, or facilitate
the exercise of, a person’s or institution’s religion.
As Justice Brennan explained, the “government may take religion into
account . . . to exempt, when possible, from generally applicable governmental
regulation individuals whose religious beliefs and practices would otherwise
thereby be infringed, or to create without state involvement an atmosphere in
which voluntary religious exercise may flourish.”

Euro-Med v. Province of Batangas, 495 SCRA 30 [2006]

Facts:
Petitioner Euro-Med Laboratories, Phil., Inc. filed a complaint against Provice of
Batangas for unpaid balance still due to the petitioner. Respondent alleged that
some payments it had already made were not reflected in the computation set forth
in the complaint and that it was continuously exerting genuine and earnest efforts
"to find out the true and actual amount owed."
At the conclusion of petitioner's presentation of evidence, respondent filed a
motion to dismiss 7 the complaint on the ground that the primary jurisdiction over
petitioner's money claim was lodged with the Commission on Audit (COA).
Respondent pointed out that petitioner's claim, arising as it did from a series of
procurement transactions with the province, was governed by the Local
Government Code provisions and COA rules and regulations on supply and
property management in local governments. Respondent argued that the case
called for a determination of whether these provisions and rules were complied
with, and that was within the exclusive domain of COA to make.

Issue:
WON it is the COA or RTC which has primary jurisdiction to pass upon
petitioner's money claim against the Province of Batangas.

Held:
We rule that it is the COA which does. Therefore, we deny the petition.
The doctrine of primary jurisdiction holds that if a case is such that its
determination requires the expertise, specialized training and knowledge of an
administrative body, relief must first be obtained in an administrative proceeding
before resort to the courts is had even if the matter may well be within their proper
jurisdiction. 10 It applies where a claim is originally cognizable in the courts and
comes into play whenever enforcement of the claim requires the resolution of
issues which, under a regulatory scheme, have been placed within the special
competence of an administrative agency. In such a case, the court in which the
claim is sought to be enforced may suspend the judicial process pending referral of
such issues to the administrative body for its view 11 or, if the parties would not
be unfairly disadvantaged, dismiss the case without prejudice. 12
This case is one over which the doctrine of primary jurisdiction clearly held sway
for although petitioner's collection suit for P487,662.80 was within the jurisdiction
of the RTC, 13 the circumstances surrounding petitioner's claim brought it clearly
within the ambit of the COA's jurisdiction.

Nisperos v. Nisperos-Ducusin (G.R. No. 189570; July 31, 2013)

CASE DIGEST: HEIRS OF SANTIAGO NISPEROS, et al. v. MARISSA


NISPEROS-DUCUSIN. (G.R. No. 189570; July 31, 2013).

FACTS: The instant case stemmed from a complaint filed by petitioners with the
DARAB alleging the following antecedents.

The 15,837-square-meter parcel of land subject of the instant case is part of the
58,350-square-meter agricultural land in Pao Sur, San Fernando City, La Union
acquired by Santiago Nisperos, the predecessor of petitioners, during his lifetime.
He declared said property for taxation purposes starting December 1947.
When Santiago and his wife Estefania died, they were survived by their nine
children. The heirs of Santiago, petitioners herein, claim that the subject property
was occupied, controlled and tilled by all nine children of Santiago. They paid
taxes for it and even hired farm workers under Maria and Ciprianas supervision for
the cultivation of the same. For taxation purposes, however, it was initially
declared only under the name of Maria.Starting 1988, it was declared under the
names of Maria and Cipriana.

During the time when Maria and Cipriana were overseeing the property, Maria
took respondent Marissa Nisperos-Ducusin, a daughter of their cousin Purita, as
her ward and raised her like her own child.

On February 12, 1988, Maria and Cipriana, acting as representatives of their other
siblings, executed a Deed of Donation Mortis Causain favor of petitioners over the
58,350-square-meter property and another 46,000-square-meter property.

On April 28, 1992, a Deed of Voluntary Land Transfer(VLT) over the subject
property was executed between Maria and Cipriana as landowners, and
respondent, who was then only 17 years old, as farmer-beneficiary. The instrument
was signed by the three in the presence of witnesses Anita, Lucia and Marcelina
Gascon and Municipal Agrarian Reform Officer Susimo Asuncion. The same was
notarized by Notary Public Atty. Roberto E. Caoayan.

On June 24, 1992, Certificate of Land Ownership Award (CLOA) No.


0002122453902was issued to respondent by the Department of Agrarian Reform
(DAR) over the subject property.

Alleging fraud on the part of respondent which petitioners claim to have


discovered only in August 2001, petitioners filed a complaint on September 6,
2001 with the Municipal Agrarian Reform Office (MARO) of San Fernando City,
La Union. Unfortunately, no settlement between petitioners and respondent was
reached prompting the MARO to issue a Certificate to File Action.

On January 23, 2002, petitioners filed with the DARAB a complaint for annulment
of documents and damages against respondent. Petitioners contended that the
transfer of ownership over the subject land was made without the consent of the
heirs of Santiago and that respondent took advantage of Marias senility and made
it appear that Maria and Cipriana sold said property by virtue of the VLT. They
further alleged that said document was falsified by respondent because Maria
could not anymore sign but could only affix her thumbmark as she did in a 1988
Deed of Donation. To support their complaint, they attached a Joint Affidavit of
Denialby Anita and Lucia Gascon the supposed instrumental witnesses to the
VLT. In said affidavit, Anita and Lucia claimed that the signatures appearing
therein are not theirs as they never affixed their signatures on said document. They
further stated that they were never aware of said document.

On October 16, 2002, DARAB Regional Adjudicator Rodolfo A. Caddarao


rendered a Decisionannulling the VLT and OCT/CLOA in respondents name.

The Regional Adjudicator noted that the land supposedly owned by Maria and
Cipriana (which includes the 15,837-square-meter subject property) has a total
area of 58,350 square meters. Considering that there are two owners, he ruled that
the individual share of each would be less than five hectares each and well within
the retention limit.

The Regional Adjudicator also held there was reason to believe that Maria and
Ciprianas names were stated in the tax declaration for purposes of taxation only as
no evidence was presented that they lawfully acquired the property from their
parents. It was also ruled that the issuance of the title in respondents name was not
in accordance with agrarian laws because she cannot be considered as a tenant but
more of an heir of the transferors.
On September 16, 2008, the DARAB rendered a Decisionreversing the decision of
the Regional Adjudicator and upholding the validity of the VLT and respondents
title.

The DARAB dismissed petitioners claim of fraud since the VLT was executed in
the presence of DAR-MARO Susimo Asuncion, signed by three instrumental
witnesses and notarized by Atty. Roberto E. Caoayan of the DAR. It likewise held
that the records are bereft of any indication that fraud was employed in the
transfer, and mere conjectures that fraud might have been exerted just because
Maria was already of advanced age while respondent was her care giver or ward is
not evidence. The DARAB also did not give credence to the Affidavit of Denial
by the instrumental witnesses since the statements there are mere hearsay because
the affiants were not cross-examined.

Aggrieved, petitioners elevated the case to the CA via a petition for reviewwhere
they raised the following issues : (1) whether the subject property is covered by
the Comprehensive Agrarian Reform Program (CARP); (2) whether the VLT is
valid having been issued through misrepresentation and fraud; and (3) whether the
action for annulment had already prescribed.

On July 13, 2009, the appellate court rendered the assailed decision dismissing the
petition for review and upholding the DARAB decision. It ruled that the Regional
Adjudicator acted with grave abuse of discretion when it held that the subject
property was no longer covered by our agrarian laws because of the retention
rights of petitioners. The CA held that retention rights, exclusion of a property
from CARP coverage and the qualification and disqualification of agrarian reform
beneficiaries are issues not cognizable by the Regional Adjudicator and the
DARAB but by the DAR Secretary. The appellate court nevertheless held that
petitioners failed to discharge their burden of proving that fraud attended the
execution of the VLT. It also agreed with the DARAB that considering a
certificate of title was already issued in favor of respondent, the same became
indefeasible and incontrovertible by the time petitioners instituted the case in
January 2002, and thus may no longer be judicially reviewed.

ISSUE: Which has jurisdiction over the complaint, the DAR Secretary or the
DARAB?

HELD: The complaint should have been lodged with the Office of the DAR
Secretary and not with the DARAB.

Section 1, Rule II of the 1994 DARAB Rules of Procedure, the rule in force at the
time of the filing of the complaint by petitioners in 2001, provides.

SECTION 1. Primary and Exclusive Original and Appellate Jurisdiction. The


Board shall have primary and exclusive jurisdiction, both original and appellate, to
determine and adjudicate all agrarian disputes involving the implementation of the
Comprehensive Agrarian Reform Program (CARP) under Republic Act No. 6657,
Executive Order Nos. 228, 229 and 129-A, Republic Act No. 3844 as amended by
Republic Act No. 6389, Presidential Decree No. 27 and other agrarian laws and
their implementing rules and regulations. Specifically, such jurisdiction shall
include but not be limited to cases involving the following.

GENERAL PROVISIONS

Rule 1 Cases:

AURORA N. DE PEDRO v. ROMASAN DEVELOPMENT CORPORATION,


GR No. 194751, 2014-11-26
Facts:
This Rule 45 petition seeks the review of the Court of Appeals July 7, 2010
decision
This case originated from separate complaints for nullification of free patent and
original certificates of title, filed against several defendants.[1] One of the
defendants is petitioner Aurora De Pedro (De Pedro).[2] The... complaints were
filed by respondent Romasan Development Corporation before the Regional Trial
Court of Antipolo City on July 7, 1998.
Respondent Romasan Development Corporation alleged in its complaints that it
was the owner and possessor of a parcel of land in Antipolo City.
Based on respondent's narrative, its representative, Mr. Rodrigo Ko, discovered
sometime in November 1996 that De Pedro put up fences on a portion of its
Antipolo property.[6] Mr. Ko confronted De Pedro regarding her acts, but she was
able to show... title and documents evidencing her ownership
Mr. Ko informed respondent about the documents.[8] Upon checking with the
Community Environment and Natural Resources Office-Department of
Environment and Natural Resources (CENRO-DENR), it was discovered that the
DENR issued free patents covering... portions of respondent's property
Based on these free patents, the Register of Deeds issued titles covering portions
of respondent's property.
Respondent further alleged in its separate complaints that the government could
not legally issue the free patents because at the time of their issuance, the land was
already released for disposition to private individuals.
Respondent also prayed for the payment of attorney's fees and exemplary
damages.[14]
Attempts to personally serve summons on De Pedro failed.[15] The officer's
return, dated February 22, 1999 reads in part:
OFFICER'S RETURN
I HEREBY CERTIFY that on the 15th and 18th day of February, 1999, I have
served a copy of the summons with complaint and annexes dated January 29, 1999
issued by Regional Trial Court, Fourth Judicial Region, Branch 74, Antipolo
City... upon defendants in the above-entitled case on the following, to wit;
1. AURORA N. DE PEDRO Unserved for the reason that according to the
messenger of Post Office of Pasig their [sic] is no person in the said given
address.[16]
Respondent filed a motion to serve summons and the complaint by publication
On August 17, 1998, the Regional Trial Court granted the motion.[18] The
summons and the complaint were published in People's Balita on its April 24, May
1, and May 8, 1998 issues.[
On July 15, 1999, respondent moved to declare all defendants in its complaints,
including De Pedro, in default for failure to file their answers.
Respondent also moved to be allowed to present evidence ex parte.[21] The
Regional Trial Court granted the motions on August 19, 1999.
On January 7, 2000, the Regional Trial Court issued an order declaring as nullity
the titles and free patents issued to all defendants in respondent's complaint,
including the free patent issued to De Pedro.
In so ruling, the Regional Trial Court noted that none of the defendants, including
De Pedro, filed an answer to respondent's complaints.[25] The Regional Trial
Court also noted the committee report admitting CENRO's irregularity in the
issuance of... the free patents to the defendants in the case.
The Regional Trial Court also found that the title and free patent issued to De
Pedro were void.[27] As early as August 30, 1937, or before the free patents were
issued to the defendants in the case, OCT No. 438 was already issued to the
property's... original owner.[28] Hence, the property was already "segregated
from the mass of public domain" that can be disposed by the government.[29]
On March 30, 2000, De Pedro, through counsel, filed before the Regional Trial
Court a motion for new trial, alleging that the counsel received notice of the
January 7, 2000 decision on March 16, 2000.
De Pedro argued that the Regional Trial Court did not acquire jurisdiction over her
person because of improper and defective service of summons. Citing the officer's
return dated February 22, 1999, De Pedro pointed out that summons was not
personally served upon her "for... the reason that according to the messenger of
Post Office of Pasig their (sic) is no person in the said given address."
De Pedro al
De Pedro also argued that the case should have been dismissed on the ground of
litis pendentia.
On September 30, 2002, the Regional Trial Court issued an order denying De
Pedro's motion for new trial.
The Regional Trial Court ruled that summons was validly served upon De Pedro
through publication, in accordance with the Rules of Court.[
Moreover, counting from the date of the summons' publication beginning on
March 2, 2000, the motion for new... trial was filed beyond the 15-day period
within which the motion may be filed.[35] Therefore, the Regional Trial Court
decision had become final and executory.
The Regional Trial Court also ruled that the reckoning period for filing the motion
for new trial cannot be De Pedro's counsel's receipt of the decision. This is
because at the time of the issuance of the court's decision, which had already
become final and executory, De
Pedro's counsel was yet to enter his appearance for De Pedro.[37]
De Pedro filed a petition for certiorari before the Court of Appeals, alleging that
the Regional Trial Court committed grave abuse of discretion when it denied her
motion for new trial.
he Court of Appeals dismissed the petition for certiorari for lack of merit,... De
Pedro's motion for reconsideration was denied in the Court of Appeals resolution
dated August 24, 2006.
De Pedro elevated the case to this court, but this was likewise denied in the
resolution dated October 4, 2006 for failure to pay the Special Allowance for the
Judiciary and sheriff's fees.
On October 11, 2006, De Pedro filed before the Court of Appeals a petition for
annulment of the January 7, 2000 judgment of the Regional Trial Court[43] on
grounds of lack of jurisdiction, litis pendentia, and for having been dispossessed of
her... property without due process.
Citing Pantaleon v. Asuncion,[44] De Pedro pointed out that "[d]ue process of law
requires personal service to support a personal judgment, and, when the
proceeding is strictly in personam brought to determine the personal rights and...
obligations of the parties, personal service within the state or a voluntary
appearance in the case is essential to the acquisition of jurisdiction [so] as to
constitute compliance with the constitutional requirement of due process."
The Court of Appeals ruled that since petitioner already availed herself of the
remedy of new trial, and raised the case before the Court of Appeals via petition
for certiorari, she can no longer file a petition for annulment of judgment.
On January 13, 2011, De Pedro filed before this court a Rule 45 petition, seeking
the reversal of the July 7, 2010 Court of Appeals decision and the December 3,
2010 Court of Appeals resolution.
Issues:
The issues in this case are:
Whether the trial court decision was void for failure of the trial court to acquire
jurisdiction over the person of petitioner Aurora N. De Pedro; and
Ruling:
Petitioner argues that respondent's prayer for attorney's fees, appearance fees,
exemplary damages, and costs of suit sought to establish personal obligations upon
petitioner in favor of respondent.[55] Hence, the case filed by respondent before
the
Regional Trial Court was an action in personam, which required personal service
upon her for the court's acquisition of jurisdiction over her person.[56] In this
case, the Regional Trial Court allowed service of summons by publication
instead... of ordering that summons be served by substituted service.[57]
Improper service of summons rendered the trial court decision null and void.[58]
It means that the court could not acquire jurisdiction over the person of...
petitioner.[59]
Petitioner also argues that respondent's complaints were dismissible on the ground
of litis pendentia, pointing to the alleged pending case between the same parties
and involving same subject matter at the time when respondent filed its complaint
before the Regional Trial
Court in 1998.[60] The alleged pending case was filed in 1997 by petitioner and
her spouse against respondent, seeking "enforce[ment] of their rights as owners,
and claim[ing] damages for the unlawful and illegal acts of dispossession,
terrorism and... violence which they, their family and their close relatives were
subjected to by [respondent]."
On her ownership of the property, petitioner argues that she was able to obtain
OCT No. P-691 in 1991 in strict and faithful compliance with all the
requirements... petitioner refers to a counter-affidavit issued by a certain Jesus
Pampellona, Deputy Public Land Inspector of CENRO-Antipolo,... Petitioner
highlights Pampellona's statements that the free patent applicants for the property
were found to be in "actual, public, adverse and continuous possession on the
specific lots applied for by them with several improvements like the house of Mrs.
Aurora de Pedro and several fruit[-]bearing trees with an average age of 20-25
years scattered within the twelve (12) hectares area applied for by the above
named applicants;"
Lastly, petitioner argues that the trial court decision was null and void, considering
that petitioner's title was cancelled in contravention of Section 48 of Presidential
Decree No. 1529, which prohibits collateral attack upon certificates of title
In its comment, respondent argues that the process server tried other forms of
substituted service, including service by registered mail.
Respondent also argues that petitioner was in evident malice and bad faith when
she allegedly did not disclose in her petition other actions taken by her after the
Regional Trial Court had denied her motion for new trial.
The sheriff's return must show the details of the efforts exerted to personally serve
summons upon defendants or respondents, before substituted service or service by
publication is availed
Courts may exercise their powers validly and with binding effect if they acquire
jurisdiction over: (a) the cause of action or the subject matter of the case; (b) the
thing or the res; (c) the parties; and (d) the remedy.
Jurisdiction over the subject matter refers to the power or authority of courts to
hear and decide cases of a general class.[73] It is conferred by the Constitution or
by law.[74] It is not acquired through administrative... issuances or court orders.
It is not acquired by agreement, stipulation, waiver,[75] or silence.[76] Any
decision by a court, without a law vesting jurisdiction upon such court, is void.
Jurisdiction over the thing or res is the power of the court over an object or thing
being litigated. The court may acquire jurisdiction over the thing by actually or
constructively seizing or placing it under the court's custody.
Jurisdiction over the parties refers to the power of the court to make decisions that
are binding on persons. The courts acquire jurisdiction over complainants or
petitioners as soon as they file their complaints or petitions. Over the persons of
defendants or... respondents, courts acquire jurisdiction by a valid service of
summons or through their voluntary submission.[78] Generally, a person
voluntarily submits to the court's jurisdiction when he or she participates in the
trial despite improper service of... summons.
Courts[79] and litigants must be aware of the limits and the requirements for the
acquisition of court jurisdiction. Decisions or orders issued by courts outside their
jurisdiction are void. Complaints or petitions filed before the wrong court... or
without acquiring jurisdiction over the parties may be dismissed.[80]
Petitioner argued that the trial court did not acquire jurisdiction over her person
because she was not properly served with summons. After the summons had
returned unserved to petitioner because "there [was] no person in the said given
address,"[81]... the trial court allowed the publication of the summons to
petitioner.
Jurisdiction over the parties is required regardless of the type of action whether the
action is in personam, in rem, or quasi in rem.
In actions in personam, the judgment is for or against a person directly.[82]
Jurisdiction over the parties is required in actions in personam because they seek
to impose personal responsibility or liability upon a person.[83]
Courts need not acquire jurisdiction over parties on this basis in in rem and quasi
in rem actions. Actions in rem or quasi in rem are not directed against the person
based on his or her personal liability.
Actions in rem are actions against the thing itself. They are binding upon the
whole world.[85] Quasi in rem actions are actions involving the status of a
property over which a party has interest.[86] Quasi... in rem actions are not
binding upon the whole world. They affect only the interests of the particular
parties.[87]
However, to satisfy the requirements of due process, jurisdiction over the parties
in in rem and quasi in rem actions is required.
An action for annulment of certificate of title is quasi in rem. It is not an action
"against a person on the basis of his personal liability,"[94] but an action that
subjects a person's interest over a property to a burden. The action... for
annulment of a certificate of title threatens petitioner's interest in the property.
Petitioner is entitled to due process with respect to that interest. The court does
not have competence or authority to proceed with an action for annulment of
certificate of... title without giving the person, in whose name the certificate was
issued all the opportunities to be heard.
Hence, regardless of the nature of the action, proper service of summons is
imperative. A decision rendered without proper service of summons suffers a
defect in jurisdiction. Respondent's institution of a proceeding for annulment of
petitioner's certificate of title... is sufficient to vest the court with jurisdiction over
the res, but it is not sufficient for the court to proceed with the case with authority
and competence.
Personal service of summons is the preferred mode of service of summons.[95]
Thus, as a rule, summons must be served personally upon the defendant or
respondent wherever he or she may be found. If the defendant or respondent
refuses to receive... the summons, it shall be tendered to him or her.
If the defendant or respondent is a domestic juridical person, personal service of
summons shall be effected upon its president, managing partner, general manager,
corporate secretary, treasurer, or in-house counsel wherever he or she may be
found.
Other modes of serving summons may be done when justified. Service of
summons through other modes will not be effective without showing serious
attempts to serve summons through personal service. Thus, the rules allow
summons to be served by substituted service only for... justifiable causes and if the
defendant or respondent cannot be served within reasonable time.[98] Substituted
service is effected "(a) by leaving copies of the summons at the defendant's
residence with some person of suitable age and discretion then... residing therein,
or (b) by leaving the copies at defendant's office or regular place of business with
some competent person in charge thereof."[99]
Service of summons by publication in a newspaper of general circulation is
allowed when the defendant or respondent is designated as an unknown owner or
if his or her whereabouts are "unknown and cannot be ascertained by diligent
inquiry."[100] It may... only be effected after unsuccessful attempts to serve the
summons personally, and after diligent inquiry as to the defendant's or
respondent's whereabouts.
Service of summons by extraterritorial service is allowed after leave of court when
the defendant or respondent does not reside or is not found in the country or is
temporarily out of the country.[101]
If a defendant or respondent voluntarily appears in trial or participates in the
proceedings, it is generally construed as sufficient service of summons.[102]
In this case, summons was served by publication.
A look into the content of the sheriff's return will determine if the circumstances
warranted the deviation from the rule preferring personal service of summons over
other modes of service. The sheriff's return must contain a narration of the
circumstances showing efforts... to personally serve summons to the defendants or
respondents and the impossibility of personal service of summons.
A sheriff's return enjoys the presumption of regularity in its issuance if it contains
(1) the details of the circumstances surrounding the sheriff's attempt to serve the
summons personally upon the defendants or respondents; and (2) the particulars
showing the impossibility of... serving the summons within reasonable time.[107]
It does not enjoy the presumption of regularity if the return was merely pro forma.
Failure to state the facts and circumstances that rendered service of summons
impossible renders service of summons and the return ineffective. In that case, no
substituted service or service by publication can be valid.
This return shows no detail of the sheriff's efforts to serve the summons personally
upon petitioner. The summons was unserved only because the post office
messenger stated that there was no "Aurora N. De Pedro" in the service address.
The return did not show that... the sheriff attempted to locate petitioner's
whereabouts. Moreover, it cannot be concluded based on the return that personal
service was rendered impossible under the circumstances or that service could no
longer be made within reasonable time.
The lack of any demonstration of effort on the part of the sheriff to serve the
summons personally upon petitioner is a deviation from this court's previous
rulings that personal service is the preferred mode of service, and that the sheriff
must narrate in his or her return the... efforts made to effect personal service.
Thus, the sheriff's return in this case was defective. No substituted service or
service by publication will be allowed based on such defective return.
The issuance of a judgment without proper service of summons is a violation of
due process rights. The judgment, therefore, suffers a jurisdictional defect. The
case would have been dismissible had petitioner learned about the case while trial
was pending. At... that time, a motion to dismiss would have been proper. After
the trial, the case would have been the proper subject of an action for annulment of
judgment.
Petitioner learned about the action for annulment of title only after trial. Instead of
filing an action for annulment of judgment, however, she filed a motion for new
trial without alleging any proper ground.
Petitioner insisted in her motion for new trial that the trial court did not acquire
jurisdiction over her person. She did not allege that fraud, accident, mistake, or
excusable negligence impaired her rights. Neither did she allege that she found
newly discovered... evidence that could have altered the trial court decision.
When her motion for new trial was denied, she filed a petition for certiorari,
insisting that her motion for new trial should have been granted on the ground of
lack of jurisdiction over her person. The Court... of Appeals denied the petition
for her failure to allege any ground for new trial. We cannot attribute error on the
part of the Court of Appeals for this denial because, indeed, lack of jurisdiction is
not a ground for granting a new trial.
What cannot be denied is the fact that petitioner was already notified of
respondent's action for annulment of petitioner's title when she filed a motion for
new trial and, later, a petition for certiorari. At that time, petitioner was deemed,
for purposes of due process,... to have been properly notified of the action
involving her title to the property. Lack of jurisdiction could have already been
raised in an action for annulment of judgment.
Thus, when petitioner erroneously filed her motion for new trial and petition for
certiorari instead of an action for annulment of judgment, she was deemed to have
voluntarily participated in the proceedings against her title. The actions and
remedies she chose to avail... bound her. Petitioner's failure to file an action for
annulment of judgment at this time was fatal to her cause. We cannot conclude
now that she was denied due process.
WHEREFORE, the petition is DENIED. The Court of Appeals July 7, 2010
decision in CA G.R. SP. No. 96471 is AFFIRMED.

COMMENCEMENT OF ACTION

RULE 2 CASES:

SECRETARY LEILA M. DE LIMA v. MAGTANGGOL B. GATDULA, GR No.


204528, 2013-02-19
Facts:
on 27 February 2012, respondent Magtanggol B. Gatdula filed a Petition for the
Issuance of a Writ of Amparo in the Regional Trial Court of Manila.
The Amparo was directed against petitioners Justice Secretary Leila M. De Lima,
Director Nonnatus R. Rojas and Deputy Director Reynaldo O. Esmeralda of the
National Bureau of Investigation (DE LIMA, ET AL. for brevity). Gatdula wanted
De Lima, et al. "to cease and desist... from framing up Petitioner [Gatdula] for the
fake ambush incident by filing bogus charges of Frustrated Murder against
Petitioner [Gatdula] in relation to the alleged ambush incident."
Instead of deciding on whether to issue a Writ of Amparo, the judge issued
summons and ordered De Lima, et al. to file an Answer.
.
He also set the case for hearing on 1 March 2012. The hearing was held allegedly
for determining whether a temporary... protection order may be issued.
ued. During that hearing, counsel for De Lima, et al. manifested that a Return, not
an Answer, is appropriate for Amparo cases.
Judge Pampilo insisted that "[s] ince no writ has been issued, return is not the
required pleading but answer".
The judge noted that the Rules of Court apply suppletorily in Amparo... cases.
He opined that the Revised Rules of Summary Procedure applied and thus
required an Answer.
Judge Pampilo proceeded to conduct a hearing on the main case on 7 March
2012.[10] Even without a Return nor an Answer, he ordered the parties to file their
respective memoranda within five (5) working days after that hearing. Since the
period to file an
Answer had not yet lapsed by then, the judge also decided that the memorandum
of De Lima, et al. would be filed in lieu of their Answer.[11]
On 20 March 2012, the RTC rendered a "Decision" granting the issuance of the
Writ of Amparo. The RTC also granted the interim reliefs prayed for, namely:
temporary protection, production and inspection orders. The production and
inspection orders were in relation... to the evidence and reports involving an on-
going investigation of the attempted assassination of Deputy Director Esmeralda.
It is not clear from the records how these pieces of evidence may be related to the
alleged threat to the life, liberty or security of the respondent
Gatdula.
the RTC denied the Motion for Reconsideration dated 23 March 2012 filed by De
Lima, et al.
Petitioners Sec. De Lima, et al. thus came to this Court assailing the RTC
"Decision" dated 20 March 2012 through a Petition for Review on Certiorari
SEC. 19. Appeal. Any party may appeal from the final judgment or order to the
Supreme Court under Rule 45. The appeal may raise questions of fact or law or
both. x x x (Emphasis supplied).
Issues:
The "Decision" dated 20 March 2012 assailed by the petitioners could not be the
judgment or final order that is appealable under Section 19 of the Rule on the Writ
of Amparo.
Ruling:
the "Decision" dated 20 March 2012 granting the writ of Amparo is not the
judgment or final order contemplated under this rule. Hence, a Petition for Review
under Rule 45 may not yet be the proper remedy at this time.
The remedy of the Writ of Amparo is an equitable and extraordinary remedy to
safeguard the right of the people to life, liberty[12] and security[13] as enshrined
in the 1987 Constitution.[14] The Rule on the Writ... of Amparo was issued as an
exercise of the Supreme Court's power to promulgate rules concerning the
protection and enforcement of constitutional rights.[15] It aims to address
concerns such as, among others, extrajudicial killings and enforced...
disappearances.[16]
It is initiated through a petition[18] to be filed in a Regional Trial Court,
Sandiganbayan, the
Court of Appeals, or the Supreme Court.
The judge or justice then makes an "immediate" evaluation[20] of the facts as
alleged in the petition and the affidavits submitted "with the attendant
circumstances detailed".
is proper if the petition and the supporting affidavits do not show t
After evaluation, the judge has the option to issue the Writ of Amparo[22] or
immediately dismiss the case. Dismissal is proper if the petition and the
supporting affidavits do not show that the petitioner's right to life, liberty or...
security is under threat or the acts complained of are not unlawful. On the other
hand, the issuance of the writ itself sets in motion presumptive judicial protection
for the petitioner. The court compels the respondents to appear before a court of
law to show whether the... grounds for more permanent protection and interim
reliefs are necessary.
The respondents are required to file a Return[23] after the issuance of the writ
through the clerk of court. The Return serves as the responsive pleading to the
petition.[24] Unlike an Answer, the Return has other purposes aside... from
identifying the issues in the case. Respondents are also required to detail the
actions they had taken to determine the fate or whereabouts of the aggrieved party.
This is clear from the tenor of the dispositive portion of the
"Decision", to wit:
The Branch Clerk of Court of Court [sic] is hereby DIRECTED to issue the Writ
of Amparo.
Likewise, the Branch Clerk of Court is hereby DIRECTED to effect the service of
the Writ of Amparo in an expeditious manner upon all concerned, and for this
purpose may call upon the assistance of any military or civilian agency of the
government.
This "Decision" pertained to the issuance of the writ under Section 6 of the Rule
on the Writ of Amparo, not the judgment under Section 18. The "Decision" is thus
an interlocutory order, as suggested by the fact that temporary protection,
production... and inspection orders were given together with the decision. The
temporary protection, production and inspection orders are interim reliefs that may
be granted by the court upon filing of the petition but before final judgment is
rendered.
The confusion of the parties arose due to the procedural irregularities in the RTC.
First, the insistence on filing of an Answer was inappropriate. It is the Return that
serves as the responsive pleading for petitions for the issuance of Writs of
Amparo. The requirement to file an Answer is contrary to the intention of the
Court to provide a speedy... remedy to those whose right to life, liberty and
security are violated or are threatened to be violated.
Judge Pampilo's basis for requiring an Answer was mentioned in his Order dated 2
March 2012:
Under Section 25 of the same rule [on the Writ of Amparo], the Rules of Court
shall apply suppletorily insofar as it is not inconsistent with the said rule.
Considering the summary nature of the petition, Section 5 of the Revised Rules of
Summary Procedure shall apply.
Section 5. Answer Within ten (10) days from service of summons, the defendant
shall file his Answer to the complaint and serve a copy thereof on the plaintiff.
this type of summary procedure only applies to MTC/MTCC/MCTCs. It is mind-
boggling how this rule could possibly apply to proceedings in an RTC. Aside from
that, this Court limited the application of summary procedure to certain civil and...
criminal cases. A writ of Amparo is a special proceeding. It is a remedy by which
a party seeks to establish a status, a right or particular fact.[34] It is not a civil nor
a criminal action, hence, the application of the Revised Rule on
Summary Procedure is seriously misplaced.
The second irregularity was the holding of a hearing on the main case prior to the
issuance of the writ and the filing of a Return.
Worse, is the trial court's third irregularity: it required a memorandum in lieu of a
responsive pleading (Answer) of De Lima, et al.
The Return in Amparo cases allows the respondents to frame the issues subject to
a hearing. Hence, it should be done prior to the hearing, not after. A memorandum,
on the other hand, is a synthesis of the claims of the party litigants and is a final
pleading usually... required before the case is submitted for decision. One cannot
substitute for the other since these submissions have different functions in
facilitating the suit.
More importantly, a memorandum is a prohibited pleading under the Rule on the
Writ of Amparo.
The fourth irregularity was in the "Decision" dated 20 March 2012 itself. In the
body of its decision, the RTC stated:
"Accordingly this court GRANTS the privilege of the writ and the interim reliefs
prayed for by the petitioner." (Emphasis supplied).
This gives the impression that the decision was the judgment since the
phraseology is similar to Section 18 of the Rule on the Writ of Amparo
The privilege of the Writ of Amparo should be distinguished from the actual order
called the Writ of Amparo. The privilege includes availment of the entire
procedure outlined in A.M. No. 07-9-12-SC, the Rule on the Writ of Amparo.
After... examining the petition and its attached affidavits, the Return and the
evidence presented in the summary hearing, the judgment should detail the
required acts from the respondents that will mitigate, if not totally eradicate, the
violation of or the threat to the petitioner's... life, liberty or security.
A judgment which simply grants "the privilege of the writ" cannot be executed. It
is tantamount to a failure of the judge to intervene and grant judicial succor to the
petitioner. Petitions filed to avail of the privilege of the Writ of Amparo arise out
of very real and... concrete circumstances. Judicial responses cannot be as
tragically symbolic or ritualistic as "granting the privilege of the Writ of Amparo."

S-ar putea să vă placă și